You are on page 1of 83

San Beda College of Law CIVIL LAW REVIEW

PROPERTY
CLASSIFICATION OF PROPERTY: Movable Property


LADERA v. HODGES
CA-G.R. No. 8027-R, 23 September 1952
[ESTEBAN]

FACTS:
Ladera entered into a contract with Hodges whereby the latter promised to
sell a lot subject to the condition that in case of failure of the purchaser to
make monthly payment within 60 days after due, the contract is considered as
rescinded and annulled. It is likewise stipulated that in such case, all sums of
money paid would be considered rentals and the vendor shall be at liberty to
dispose of the parcel of land with all the improvements theron to any other
person in a manner as if the contract had never been made. After the
execution of the contract, Ladera built on a lot a house of mixed materials
assessed at P4500. Unfortunately, Ladera failed to pay the agreed
installments, whereupon the appellant rescinded the contract and filed an
action for ejectment.
The MTC rendered a decision upon agreement of the parties Ladera to
vacate and surrender possession of the lot and pay P10 a month until delivery
of the premises. The court issued an alias writ of execution and pursuant
thereto the sheriff levied upon all rights, interests, and participation over your
house standing on the lot. The sheriff posted the notices of the sale but did
not publish the same in a newspaper of general circulation. At the auction sale
Ladera did not attend because she had gone to Manila and the sheriff sold the
property to Avelina Magno as the highest bidder. On July 6, 1948, Hodges
sold the lot to Manuel Villa and on the same day the latter purchased the
house from Magno for P200 but this last transaction was not recorded.
Ladera returned to Iloilo after the sale and learned of its results. She went to
see the sheriff and upon the latters representation that she could redeem the
property, she paid him P230 and the sheriff issued a receipt. It does not
appear, however, that this money was turned over to Hodges. Thereupon,
Spouses Ladera filed an action against Hodges, the sheriff, and the judgment
sale purchasers, Magno and Villa to set aside the sale and recover the house.
The lower court ruled in favor of Ladera. Hodges et al contend that the house
being built on land owned by another person should be regarded in law as
movable or personal property.
ISSUE: Should the house built on land owned by another be regarded as
immovable property?

1 4S 2014-2015

San Beda College of Law CIVIL LAW REVIEW



RULING:
YES. According to Article 334 of the Civil Code (now 415), immovable
property are the following: xxx Lands, building, roads, and constructions of all
kinds adhering to the soil; xxx. Applying the principle ubi lex non distinguit
nec nos distinguere debemos (when the law does not distinguish we ought
not to distinguish), the law makes no distinction as to whether the owner of
the land is or is not the owner of the building.
In the case of immovables by destination, the Code requires that they be
placed by the owner of the tenement, in order to acquire the same nature or
consideration of real property. In cases of immovable by incorporation, the
Code nowhere requires that the attachment or incorporation be made by the
owner of the land. The only criterion is union or incorporation with the soil. A
true building (not merely superimposed on the soil) is immovable or real
property, whether it is erected by the owner of the land or by usufructuary or
lessee.
(Note: Due to the unavailability of the original text via the internet, the digest above was
sourced from several digests of the original case. Rest assured that due diligence was
exercised to verify everything provided herein.)

MINDANAO BUS COMPANY v. THE CITY ASSESSOR & TREASURER


and the BOARD OF TAX APPEALS of Cagayan de Oro City
G.R. No. L-17870, 29 September 1962
[FERNANDEZ]
FACTS:
Petitioner is a public utility solely engaged in transporting passengers and
cargoes by motor trucks. It owns a land where it maintains and operates a
garage for its TPU motor trucks; a repair shop; blacksmith and carpentry
shops, and with machineries placed therein, its TPU trucks are made; body
constructed; and same are repaired in a condition to be serviceable in the
TPU land transportation business it operates.
The machineries have never been or were never used as industrial equipment
to produce finished products for sale, nor to repair machineries, parts and the
like offered to the general public indiscriminately for business or commercial
purposes.
Respondent City Assessor of Cagayan de Oro City assessed at P4,400
petitioner's above-mentioned equipment. Petitioner appealed the assessment
to the respondent Board of Tax Appeals on the ground that the same are not
realty. The Board of Tax Appeals of the City sustained the city assessor, so
petitioner herein filed with the Court of Tax Appeals a petition for the review of
the assessment.
The CTA held the petitioner liable to the payment of the realty tax on its
maintenance and repair equipment mentioned above. Hence, this petition.
2 4S 2014-2015

San Beda College of Law CIVIL LAW REVIEW



ISSUE: Should the tools and equipment in the petitioner companys repair shop
be considered immovable taxable real properties?
RULING:
NO. Movable equipment to be immobilized in contemplation of the law must
first be "essential and principal elements" of an industry or works without
which such industry or works would be "unable to function or carry on the
industrial purpose for which it was established." The tools and equipment in
question are not essential and principle municipal elements of petitioner's
business of transporting passengers and cargoes by motor trucks. They are
merely incidentals acquired as movables and used only for expediency to
facilitate and/or improve its service. The transportation business could be
carried on without the repair or service shop if its rolling equipment is repaired
or serviced in another shop belonging to another.
MAKATI LEASING AND FINANCE CORPORATION v. WEAREVER
TEXTILE MILLS, INC.
G.R. No. L-58469, 16 May 1983
[MADRILENO]
FACTS:
Private respondent Wearever Textile Mills, Inc., discounted and assigned
several receivables with Makati Leasing and Finance Corporation under a
Receivable Purchase Agreement. To secure the collection of the receivables
assigned, private respondent executed a Chattel Mortgage over certain raw
materials inventory as well as a machinery described as an Artos Aero Dryer
Stentering Range.
Upon private respondent's default, petitioner filed a petition for extrajudicial
foreclosure of the properties mortgage to it. However, the Deputy Sheriff
assigned to implement the foreclosure failed to gain entry into private
respondent's premises and was not able to effect the seizure of the
aforedescribed machinery.
Petitioner filed a complaint for judicial foreclosure for which the court issued a
writ of seizure, the enforcement of which was however subsequently
restrained upon private respondent's filing of a motion for reconsideration. On
July 13, 1981, the sheriff enforcing the seizure order, repaired to the premises
of private respondent and removed the main drive motor of the subject
machinery.
The Court of Appeals reversed the ruling of the trial court and said that the
machinery in suit cannot be the subject of replevin, much less of a chattel
mortgage, because it is a real property pursuant to Article 415 of the new Civil
Code, the same being attached to the ground by means of bolts and the only
way to remove it from respondent's plant would be to drill out or destroy the
concrete floor, the reason why all that the sheriff could do to enforce the writ
was to take the main drive motor of said machinery.
3 4S 2014-2015

San Beda College of Law CIVIL LAW REVIEW



ISSUE: Is the property in question a real or personal property
RULING:
PERSONAL PROPERTY. It must be pointed out that the characterization of
the subject machinery as chattel by the private respondent is indicative of
intention and impresses upon the property the character determined by the
parties. As stated in Standard Oil Co. of New York v. Jaramillo, 44 Phil. 630, it
is undeniable that the parties to a contract may by agreement treat as
personal property that which by nature would be real property, as long as no
interest of third parties would be prejudiced thereby.
If a house of strong materials, like what was involved in the case of Tumalad
v. Vicencio, may be considered as personal property for purposes of
executing a chattel mortgage thereon as long as the parties to the contract so
agree and no innocent third party will be prejudiced thereby, there is
absolutely no reason why a machinery, which is movable in its nature and
becomes immobilized only by destination or purpose, may not be likewise
treated as such. This is really because one who has so agreed is estopped
from denying the existence of the chattel mortgage.
SANTOS EVANGELISTA V. ALTO SURETY & INSURANCE CO. INC.
G.R. No. L-11139, 23 April 1958
[MAGULTA]
FACTS:
On June 4, 1949, Santos Evangelista instituted a civil case (entitled Santos
Evangelista v. Ricardo Rivera) for a sum of money against Ricardo Rivera. On
the same day, he obtained a writ of attachment, which levied upon a house
built by Rivera on a land leased to him. In due course, judgment was rendered
in favor of Evangelista who bought the house at public auction held in
compliance with the writ of execution. The corresponding definite deed of sale
was issued to him on on October 22, 1952, upon expiration of the period of
redemption. When Evangelista sought to take possession of the house,
Rivera refused to surrender it upon the ground that he leased the property
from the Alto Surety & Insurance Co., Inc. and that the latter is now the true
owner of said property.
After due trial, the Court of First Instance of Manila rendered judgment in favor
of Evangelista. The decision was reversed by the CA upon the ground that
although the writ of attachment in favor of Evangelista had been filed with the
Register of Deeds prior to the sale in favor of respondent, Evangelista did not
acquire thereby a preferential lien, the attachment having been levied as if the
house in questIon were immovable property, although in the opinion of the
CA, it is ostensibly a personal property. As such, the Court of Appeals held,
"the order of attachment . . . should have been served in the manner provided
in subsection (e) of section 7 of Rule 59," of the Rules of Court
ISSUE: Should a house built on a leased property by the lessee be considered
as movable or personal property for the purpose of attachment?
4 4S 2014-2015

San Beda College of Law CIVIL LAW REVIEW



RULING:
NO. The house is not personal property, much less a debt, credit or other
personal property not capable of manual delivery, but immovable property. As
explicitly held, in Laddera v. Hodges, a true building (not merely
superimposed on the soil) is immovable or real property, whether it is erected
by the owner of the land or by usufructury or lessee.
It is true that the parties to a deed of chattel mortgage may agree to consider
a house as personal property for purposes of said contract. However, this
view is good only insofar as the contracting parties are concerned. It is based,
partly, upon the principle of estoppel. Neither this principle, nor said view, is
applicable to strangers to said contract. Much less is it in point where there
has been no contract whatsoever, with respect to the status of the house
involved, as in the case at bar.
TSAI v. HON. COURT OF APPEALS
G.R. No. 120098, 2 October 2001
[MANRIQUE]
FACTS:
Respondent Ever Textile Mills, Inc. (EVERTEX) obtained a loan from
petitioner Philippine Bank of Communications (PBCom). As security for the
loan, EVERTEX executed in favor of PBCom, a deed of Real and Chattel
Mortgage over the lot under TCT No. 372097, where its factory stands, and
the chattels located therein as enumerated in a schedule attached to the
mortgage contract. On April 23, 1979, PBCom granted a second loan to
EVERTEX. The loan was secured by a Chattel Mortgage over personal
properties enumerated in a list attached thereto. These listed properties were
similar to those listed in the first mortgage deed. Afterwards, EVERTEX
purchased various machines and equipments.
On November 19, 1982, due to business reverses, EVERTEX filed for
insolvency. All its assets were taken into the custody of the Insolvency Court,
including the collateral, real and personal, securing the two mortgages. In the
meantime, PBCom, commenced extrajudicial foreclosure proceedings against
EVERTEX. On March 7, 1984, PBCom consolidated its ownership over the lot
and all the properties in it. It leased the entire factory premises to petitioner
Ruby L. Tsai and sold the factory, lock, stock and barrel to her, including the
contested machineries.
EVERTEX filed a complaint for annulment of sale, reconveyance, and
damages against PBCom, alleging inter alia that the extrajudicial foreclosure
of subject mortgage was not valid, and that PBCom, without any legal or
factual basis, appropriated the contested properties which were not included
in the Real and Chattel Mortgage of the first mortgage contract nor in the
second contract which is a Chattel Mortgage, and neither were those
properties included in the Notice of Sheriff's Sale.

5 4S 2014-2015

San Beda College of Law CIVIL LAW REVIEW



ISSUE: Are the controverted parties movable properties?
RULING:
The intent of the parties must be looked at. While it is true that the
controverted properties appear to be immobile, a perusal of the contract of
Real and Chattel Mortgage executed by the parties herein gives a contrary
indication. In the case at bar, both the trial and the appellate courts reached
the same finding that the true intention of PBCOM and the owner, EVERTEX,
is to treat machinery and equipment as chattels. As ruled by the appellate
court:
As stressed upon by appellees, appellant bank treated the machineries as
chattels; never as real properties. Indeed, the 1975 mortgage contract, which
was actually real and chattel mortgage, militates against appellants' posture. It
should be noted that the printed form used by appellant bank was mainly for
real estate mortgages. But reflective of the true intention of appellant PBCOM
and appellee EVERTEX was the typing in capital letters, immediately
following the printed caption of mortgage, of the phrase "real and chattel." So
also, the "machineries and equipment" in the printed form of the bank had to
be inserted in the blank space of the printed contract and connected with the
word "building" by typewritten slash marks. Now, then, if the machineries in
question were contemplated to be included in the real estate mortgage, there
would have been no necessity to ink a chattel mortgage specifically
mentioning as part III of Schedule A a listing of the machineries covered
thereby. It would have sufficed to list them as immovables in the Deed of Real
Estate Mortgage of the land and building involved.
As regards the 1979 contract, the intention of the parties is clear and beyond
question. It refers solely to chattels. The inventory list of the mortgaged
properties is an itemization of sixty-three (63) individually described
machineries while the schedule listed only machines and 2,996,880.50 worth
of finished cotton fabrics and natural cotton fabrics.
SERG'S PRODUCTS, INC v. PCI LEASING AND FINANCE
G.R. No. 137705, 22 August 2000
[MATEO]
FACTS:
Respondent PCI Leasing and Finance, Inc, filed with the RTC-QC a complaint
for a sum of money with an application for a writ of replevin. Respondent
Judge issued a writ of replevin directing its sheriff to seize and deliver the
machineries and equipment to PCI after 5 days and upon the payment of the
necessary expenses. In the implementation of the said writ, the sheriff
proceeded to petitioners factory, seized one machinery with word that he
would return for the other.
Petitioner asserted that properties sought to be seized were immovable as
defined in Article 415 of the Civil Code. The motion was opposed by PCI
Leasing, on the ground that the properties were still personal and therefore
6 4S 2014-2015

San Beda College of Law CIVIL LAW REVIEW



still subject to seizure and a writ of replevin. In its decision on the original
action for certiorari filed by the Petitioner, the appellate court, Citing the
Agreement of the parties, held that the subject machines were personal
property, and that they had only been leased, not owned, by petitioners; and
ruled that the "words of the contract are clear and leave no doubt upon the
true intention of the contracting parties."
ISSUE: Did the machineries become real property by virtue of immobilization?
RULING:
NO. The machines are personal property and they are proper subjects of the
Writ of Replevin. In the present case, the machines that were the subjects of
the Writ of Seizure were placed by petitioners in the factory built on their own
land. They were essential and principal elements of their chocolate-making
industry. Hence, although each of them was movable or personal property on
its own, all of them have become immobilized by destination because they
are essential and principal elements in the industry. However, contracting
parties may validly stipulate that a real property be considered as personal.
The Agreement between the parties provides The PROPERTY is, and shall
at all times be and remain, personal property notwithstanding that the
PROPERTY or any part thereof may now be, or hereafter become, in any
manner affixed or attached to or embedded in, or permanently resting upon,
real property or any building thereon, or attached in any manner to what is
permanent. After agreeing to such stipulation, they are consequently
estopped from claiming otherwise. Under the principle of estoppel, a party to a
contract is ordinarily precluded from denying the truth of any material fact
found therein. It should be stressed, however, that the Courts holding-that
the machines should be deemed personal property pursuant to the Lease
Agreement-is good only insofar as the contracting parties are concerned.
Hence, while the parties are bound by the Agreement, third persons acting in
good faith are not affected by its stipulation characterizing the subject
machinery as personal.
BURGOS, SR. v. CHIEF OF STAFF
G.R. No. L-64261, 26 December 1984
[MEDINA]
FACTS:
Assailed in this petition for certiorari prohibition and mandamus with
preliminary mandatory and prohibitory injunction is the validity of two [2]
search warrants issued by respondent Judge Ernani Cruz-Pano, Executive
Judge of the then Court of First Instance of Rizal [Quezon City], under which
the premises known as No. 19, Road 3, Project 6, Quezon City, and 784 Units
C & D, RMS Building, Quezon Avenue, Quezon City, business addresses of
the "Metropolitan Mail" and "We Forum" newspapers, respectively, were
searched, and office and printing machines, equipment, paraphernalia, motor
vehicles and other articles used in the printing, publication and distribution of
the said newspapers, as well as numerous papers, documents, books and
other written literature alleged to be in the possession and control of petitioner
7 4S 2014-2015

San Beda College of Law CIVIL LAW REVIEW



Jose Burgos, Jr. publisher-editor of the "We Forum" newspaper, were seized.
Petitioner avers, among others, that the seizure of the properties mentioned
above amounts to seizure of real properties, which cannot be validly
conducted under the strength of a search warrant. It must be noted that real
properties are not susceptible of confiscation under a search warrant. Hence
this appeal which assails the validity of the search and the seizure of the
properties of the petitioner.
ISSUE: Are the machineries seized under the disputed warrants real
properties?
RULING:
NO. Under Article 415[5] of the Civil Code of the Philippines, "machinery,
receptables, instruments or implements intended by the owner of the
tenement for an industry or works which may be carried on in a building or on
a piece of land and which tend directly to meet the needs of the said industry
or works" are considered immovable property. In Davao Sawmill Co. v.
Castillo where this legal provision was invoked, the Supreme Court ruled that
machinery which is movable by nature becomes immobilized when placed by
the owner of the tenement, property or plant, but not so when placed by a
tenant, usufructuary, or any other person having only a temporary right,
unless such person acted as the agent of the owner. In the case at bar,
petitioners do not claim to be the owners of the land and/or building on which
the machineries were placed. This being the case, the machineries in
question, while in fact bolted to the ground remain movable property
susceptible to seizure under a search warrant.
LOPEZ v. OROSA, JR. and PLAZA THEATER INC.
G.R. Nos. L-10817-18, 28 February 1958
[MENESES]
FACTS:
Petitioner Enrique Lopez agreed to supply the lumber necessary for the
construction of Vicente Orosas theater, the latter assuming personal liability
for any account that the said construction might incur. Pursuant to the verbal
agreement, Lopez delivered the lumber which was used for the construction
of the Plaza Theater on the land which Plaza Theater Inc. owned. The total
cost of the materials was not paid. After several demands for payment of the
balance and as the obligation still remained unsettled, Lopez filed a complaint
with the CFI of Batangas against Orosa and Plaza Theater, Inc. In his
complaint, Lopez prayed that in case of failure to pay the remaining balance
for the materials, the building and the land on which the theater was
constructed should be sold at a public auction and that the proceeds thereof
be applied to said indebtedness. The lower court ruled in favor of Lopez,
stating that Lopez acquired a materialmans lien over the building but such
lien did not extend to the land on which the construction was made. Lopez
tried to secure a modification of the decision in so far as the lower court
declared that the lien did not extend to the land. He contended that that the
lien created in favor of the furnisher of the materials used for the construction,
8 4S 2014-2015

San Beda College of Law CIVIL LAW REVIEW



repair or refection of a building is also extended to the land. He cited Art. 1923
of the Spanish Civil Code which reads as follows:
Art. 1923. With respect to determinate real property and real rights of the debtor, the following
are preferred:
5. Credits for refection, not entered or recorded, with respect to the real estate upon which
the refection was made, and only with respect to other credits

Lopez argued that in view of the employment of the phrase real estate, and
inasmuch as said provision does not contain any specification delimiting the
lien to the building, said article must be construed as to embrace both the land
and the building or structure adhering thereto.
ISSUE: Is a building considered an immovable property and therefore included
in the term real estate, as contended by Lopez?
RULING:
YES. The inclusion of the term building, separate and distinct from the land, in
the enumeration of what may constitute real properties under Art. 415 of the
Civil Code, could mean only one thing that a building is by itself an
immovable property. In the absence of any specific provision of law to the
contrary, a building is an immovable property, irrespective of whether or not
said structure and the land on which it is adhered to belong to the same
owner.
YAP v. TANADA
G.R. No. L-32917, 18 July 1988
[ORSUA]
FACTS:
Respondent Goulds Pumps International Inc. filed a complaint against Yap
and his wife, seeking recovery of Pl,459.30 representing the balance of the
price and installation cost of a water pump in the latter's premises. The City
Court decided in respondents favor. Hence, Yap appealed to the CFI. CFI
Judge Tanada also ruled in favor of respondent. Judge Tanada issued an
order granting Goulds' Motion for Issuance of Writ of Execution. Judge
Tanada denied Yap's motion for reconsideration and authorized execution of
the judgment. In the meantime the Sheriff levied on the water pump in
question, and by notice scheduled the execution sale thereof. But in view of
the pendency of Yap's motion for reconsideration, suspension of the sale was
directed by the judge. It appears however that a copy of this order was not
transmitted to the Sheriff "through oversight, inadvertence and pressure of
work" of the Branch Clerk of Court. So the Deputy Provincial Sheriff went
ahead with the scheduled auction sale and sold the property levied on to
Goulds as the highest bidder.
Yap filed a motion to set aside execution sale and to quash alias writ of
execution. He argued that the sale was made without the notice required by
Sec. 18, Rule 39, of the New Rules of Court, i.e., notice by publication in case
9 4S 2014-2015

San Beda College of Law CIVIL LAW REVIEW



of execution sale of real property, the pump and its accessories being
immovable because attached to the ground with character of permanency
(Art. 415, Civil Code). Yap's motion was thereafter denied. Judge Tanada
pointed out that the motion had "become moot and academic" since the
decision received by the defendant had long become final when the order for
the issuance of a writ of execution was promulgated.
ISSUE: Is the water pump involved an immovable property as what petitioner
claimed as a basis for annulling the execution sale?
RULING:
NO. The Civil Code considers as immovable property, among others, anything
attached to an immovable in a fixed manner, in such a way that it cannot be
separated therefrom without breaking the material or deterioration of the
object. The pump does not fit this description. It could be, and was in fact
separated from Yaps premises without being broken or suffering
deterioration. Obviously the separation or removal of the pump involved
nothing more complicated than the loosening of bolts or dismantling of other
fasteners.
MACHINERY AND ENGINEERING SUPPLIES, INC v. COURT OF
APPEALS,
G.R. No. L-7057, 29 October 1954
[PAGAYANAN]
FACTS:
Petitioner filed a complaint for replevin in CFI of Manila for the recovery of the
machineries and equipment delivered to Ipo Limestone Co, Inc. at their
factory in barrio Bigti, Norzagaray, Bulacan. Upon application ex-parte of the
petitioner, accompanied by the affidavit of Ramon S. Roco, president of
petitioner company, and upon approval of petitioner's bond, respondent judge
issued an order, commanding the Provincial Sheriff of Bulacan to seize and
take immediate possession of the properties specified in the order. Two
deputy sheriffs of Bulacan, and other technical men and laborers proceeded
to Bigti, for the purpose of carrying the court's order into effect. Leonardo
Contreras, manager of the respondent Company, and Pedro Torres, in charge
thereof, met the deputy sheriffs, and Contreras handed to them a letter
addressed to Atty. Palad, protesting against the seizure of the properties in
question, on the ground that they are not personal properties. Contending that
the Sheriff's duty is merely ministerial, the deputy sheriffs, Roco, the latter's
crew of technicians and laborers, Contreras and Torres, went to the factory.
Roco's attention was called to the fact that the equipment could not possibly
be dismantled without causing damages or injuries to the wooden frames
attached to them. As Roco insisted in dismantling the equipment on his own
responsibility, alleging that the bond was posted for such eventuality, the
deputy sheriffs directed that some of the supports thereof be cut.

10 4S 2014-2015

San Beda College of Law CIVIL LAW REVIEW



ISSUE: Can the machineries, which could not be dismantled without causing
damages or injuries to the frames attached, be the proper subjects of a writ of
replevin?
RULING:
NO. When the sheriff repaired to the premises of respondent, Ipo Limestone
Co., Inc., the machinery and equipment in question appeared to be attached
to the land, particularly to the concrete foundation of said premises, in a fixed
manner, in such a way that the former could not be separated from the latter
"without breaking the material or deterioration of the object." Hence, in order
to remove said outfit, it became necessary, not only to unbolt the same, but,
also, to cut some of its wooden supports. Moreover, said machinery and
equipment were "intended by the owner of the tenement for an industry"
carried on said immovable and tended "directly to meet the needs of the said
industry." For these reasons, they were already immovable property pursuant
to paragraphs 3 and 5 of Article 415 of Civil Code of the Philippines, which are
substantially identical to paragraphs 3 and 5 of Article 334 of the Civil Code of
Spain. As such immovable property, they were not subject to replevin.

CLASSIFICATION OF PROPERTY: Property in Relation


to Whom it Belongs

SALVADOR H. LAUREL v. RAMON GARCIA
G.R. No. 92013, 25 July 1990
[PARUNGAO]
FACTS:
The properties and the capital goods and services procured by the Philippines
under the Reparation Agreement for National Development projects are part
of the indemnification to the Filipino people for their losses in life and property
and their suffering during World War II. The Roppongi property consists of the
land and building specifically designated under the Agreement for the
chancery of the Philippine Embassy.
Due to the failure of the Philippine Government to provide necessary funds,
the property has remained undeveloped. The Executive Branch has been
pushing, with great vigor, its decision to sell the reparation properties starting
with the Roppongi lot.
ISSUEs:
1. Can the Roppongi property and others of its kind be alienated by the
Philippine Government?
2. Has the Roppongi property become patrimonial?
RULING:
1. As property of public dominion, the Roppongi lot is outside the commerce
of man. It cannot be alienated. Its ownership is a special collective ownership
11 4S 2014-2015

San Beda College of Law CIVIL LAW REVIEW



for general use and enjoyment, an application to the satisfaction of collective
needs, and resides in the social group. The purpose is not to serve the State
as a juridical person, but the citizens; it is intended for the common and public
welfare and cannot be the object of appropriation. The Roppongi property is
correctly classified under paragraph 2 of Article 420 as property belonging to
the State and intended for some public service.
2. The fact that the Roppongi site has not been used for a long time for actual
Embassy service does not automatically convert it to patrimonial property.
Any such conversion happens only if the property is withdrawn from public
use. A property continues to be part of the public domain, not available for
private appropriation or ownership until there is a formal declaration on the
part of the government to withdraw it from being such.
RABUCO v. VILLEGAS
G.R. No. L-24661, 28 February1974
[PULMA]
FACTS:
The Court herein upholds the constitutionality of Republic Act 3120 on the
strength of the established doctrine that the subdivision of communal land of
the State (although titled in the name of the municipal corporation) and
conveyance of the resulting subdivision lots by sale on installment basis to
bona fide occupants by Congressional authorization and disposition does not
constitute infringements of the due process clause or the eminent domain
provisions of the Constitution but operates simply as a manifestation of the
legislature's right of control and power to deal with State property.
The petitioners are the tenants of the land in dispute who are being ejected on
account of Republic Act 3120. In Republic Act 3120 Congress converted the
lot in question together with another lot in San Andres, Malate "which are
reserved as communal property" into "disposable or alienable lands of the
State to be placed under the administration and disposal of the Land Tenure
Administration" for subdivision into small lots not exceeding 120 square
meters per lot for sale on installment basis to the tenants or bona fide
occupants thereof. The respondent officials also contend that the act is
unconstitutional for depriving the City of Manila of the lots in question without
just compensation.
ISSUE: Is R.A. 3120 Constitutional?

RULING:
YES. Respondents city officials' contention that the Act must be stricken down
as unconstitutional for depriving the city of Manila of the lots in question and
providing for their sale in subdivided small lots to bona fide occupants or
tenants without payment of just compensation is untenable and without basis,
since the lots in question are manifestly owned by the city in its public and
governmental capacity and are therefore public property over which Congress
had absolute control as distinguished from patrimonial property owned by it in
12 4S 2014-2015

San Beda College of Law CIVIL LAW REVIEW



its private or proprietary capacity of which it could not be deprived without due
process and without just compensation.
Here, Republic Act 3120 expressly declared that the properties were
"reserved as communal property" and ordered their conversion into
"disposable and alienable lands of the State" for sale in small lots to the bona
fide occupants thereof. It is established doctrine that the act of classifying
State property calls for the exercise of wide discretionary legislative power
which will not be interfered with by the courts.
The Court therein reaffirmed the established general rule that "regardless of
the source or classification of land in the possession of a municipality,
excepting those acquired with its own funds in its private or corporate
capacity, such property is held in trust for the State for the benefit of its
inhabitants, whether it be for governmental or proprietary purposes. It holds
such lands subject to the paramount power of the legislature to dispose of the
same, for after all it owes its creation to it as an agent for the performance of a
part of its public work, the municipality being but a subdivision or
instrumentality thereof for purposes of local administration. Accordingly, the
legal situation is the same as if the State itself holds the property and puts it to
a different use" and stressed that "the property, as has been previously
shown, was not acquired by the City of Manila with its own funds in its private
or proprietary capacity. That it has in its name a registered title is not
questioned, but this title should be deemed to be held in trust for the State as
the land covered thereby was part of the territory of the City of Manila granted
by the sovereign upon its creation."
MACASIANO v. DIOKNO
G.R. No. 97764, 10 August 1992
[REMIGIO]
FACTS:
On June 13, 1990, the Municipality of Paranaque passed Ordinance No. 86,
Series of 1990 which authorized the closure of J. Gabriel, G.G. Cruz,
Bayanihan, Lt. Garcia Extension and Opena Streets located at Baclaran,
Paraaque, Metro Manila and the establishment of a flea market thereon. The
said ordinance was approved by the municipal council pursuant to MMC
Ordinance No. 2, Series of 1979, authorizing and regulating the use of certain
city and/or municipal streets, roads and open spaces within Metropolitan
Manila as sites for flea market and/or vending areas, under certain terms and
conditions.
On June 20, 1990, the municipal council of Paraaque issued a resolution
authorizing Paraaque Mayor Walfrido N. Ferrer to enter into contract with
any service cooperative for the establishment, operation, maintenance and
management of flea markets and/or vending areas. On August 8, 1990,
respondent municipality and respondent Palanyag, a service cooperative,
entered into an agreement whereby the latter shall operate, maintain and
manage the flea market in the aforementioned streets with the obligation to
13 4S 2014-2015

San Beda College of Law CIVIL LAW REVIEW



remit dues to the treasury of the municipal government of Paraaque.
Consequently, market stalls were put up by respondent Palanyag on the said
streets.
ISSUE: Is an ordinance or resolution Issued by the municipal council of
Paraaque authorizing the lease and use of public streets or thoroughfares as
sites for flea markets valid?
RULING:
Article 424 of the Civil Code lays down the basic principle that properties of
public dominion devoted to public use and made available to the public in
general are outside the commerce of man and cannot be disposed of or
leased by the local government unit to private persons. Properties of the local
government which are devoted to public service are deemed public and are
under the absolute control of Congress. Hence, local governments have no
authority whatsoever to control or regulate the use of public properties unless
specific authority is vested upon them by Congress. When it is already
withdrawn from public use, the property then becomes patrimonial property of
the local government unit concerned. It is only then that the respondent
municipality can "use or convey them for any purpose for which other real
property belonging to the local unit concerned might be lawfully used or
conveyed" in accordance with the last sentence of Section 10, Chapter II of
Blg. 337, known as Local Government Code.
REPUBLIC v. COURT OF APPEALS
G.R. No. 100709, 14 November 1997
[RAMPAS]
FACTS:
Josefina Morato, private respondent, applied sometime in 1972 a free patent
on a parcel of land, it was approved and it specifically stated that the same
cannot be sold or encumbered for 5 years. It was however leased and later
on, it was discovered that part of the land is a foreshore land. Petitioner
moved for the cancellation of the free patent.
ISSUE: Is the questioned land a foreshore land and thus must be reverted to
the public domain?
RULING:
YES. When the sea moved towards the estate and the tide invaded it, the
invaded property became foreshore land and passed the realm of the public
domain and accordingly cannot be a subject of a free patent. (Article 420)

14 4S 2014-2015

San Beda College of Law CIVIL LAW REVIEW



PROVINCE OF ZAMBOANGA DEL NORTE v. CITY OF ZAMBOANGA, et
al.
L-24440, 28 March 1968
[SABILALA]
FACTS:
After Zamboanga Province was divided into two (Zamboanga del Norte and
Zamboanga del Sur), Republic Act 3039 was passed providing that
All buildings, properties, and assets belonging to the former province of
Zamboanga and located within the City of Zamboanga are hereby transferred
free of charge in favor of the City of Zamboanga.''
Suit was brought alleging that this grant without just compensation was
unconstitutional because it deprived the province of property without due
process. Included in the properties were the capital site and capitol building,
certain school sites, hospital and leprosarium sites, and high school
playgrounds.
ISSUES:
1. Are the properties mentioned, properties for public use or patrimonial?
2. Should the city pay for said properties?
RULING:
1. If we follow the Civil Code classification, only the high school playgrounds
are for public use (in the sense that generally, they are available to the
general public), and all the rest are PATRIMONIAL (since they are not
devoted to public use but to public service; since they are not for public use,
under Art. 424 of the Civil Code, they are patrimonial. [NOTE: For public use if
ANYBODY can use; for public service if only AUTHORIZED persons can
use.].
[NOTE: Had they been owned by the STATE, they would not have been
patrimonial but would have been properties of public dominion for this
would include public service, conformably with Art. 420, par. 2.].
BUT if we follow the law of Municipal Corporations (and not the Civil Code),
as long as the purpose is for a public service (governmental service like public
education, public health, local administration), the property should be
considered for PUBLIC USE.
2. If the Civil Code classification is used, since almost all the properties
involved are patrimonial, the law would be unconstitutional since the province
would be deprived of its own property without just compensation.
If the law on Municipal Corporations would be followed, the properties would
be of public dominion, and therefore NO COMPENSATION would be
required.

15 4S 2014-2015

San Beda College of Law CIVIL LAW REVIEW



It is this law on Municipal Corporations that should be followed. Firstly, while
the Civil Code may classify them as patrimonial, they should not be regarded
as ordinary private property. They should fall under the control of the State,
otherwise certain governmental activities would be impaired. Secondly, Art.
424, 2nd paragraph itself says without prejudice to the provisions (or
PRINCIPLES) of special laws.
CHAVEZ v. PUBLIC ESTATES AUTHORITY
G.R. No. 133250, July 9, 2002
[SANTILLAN]
FACTS:
President Marcos issued Presidential Decree No. 1084 creating PEA. PD No.
1084 tasked PEA to reclaim land, including foreshore and submerged areas,
and to develop, improve, acquire, x x x lease and sell any and all kinds of
lands. Then President Aquino issued Special Patent, granting and
transferring to PEA the parcels of land so reclaimed under the Manila-Cavite
Coastal Road and Reclamation Project (MCCRRP). Subsequently, the
Register of Deeds issued Transfer Certificates of Title in the name of PEA,
covering the three reclaimed islands known as the Freedom Islands. PEA
entered into a Joint Venture Agreement with AMARI, a private corporation, to
develop the Freedom Islands. Under the amended JVA it required the
reclamation of an additional 250 hectares of submerged areas surrounding
these islands and PEA is obligated to transfer to AMARI the latters seventy
percent proportionate share in the reclaimed areas as the reclamation
progresses. PEA and AMARI entered into the JVA through negotiation without
public bidding. The JVA was subsequently approved, hence, this appeal to
assail the constitutionality of the JVA.
ISSUE: Can a private corporation acquire and own reclaimed foreshore and
submerged areas in Manila Bay?
RULING:
The JVA contravenes Section 3, Article XII of the 1987 Constitution which
provides that private corporations shall not hold such alienable lands of the
public domain except by lease. The transfer of title and ownership to AMARI
clearly means that AMARI will hold the reclaimed lands other than by lease.
Furthermore, since the Amended JVA also seeks to transfer to AMARI
ownership of 290.156 hectares of still submerged areas of Manila Bay, such
transfer is void for being contrary to Section 2, Article XII of the 1987
Constitution which prohibits the alienation of natural resources other than
agricultural lands of the public domain
The Regalian doctrine is deeply implanted in our legal system. Foreshore and
submerged areas form part of the public domain and are inalienable. Lands
reclaimed from foreshore and submerged areas also form part of the public
domain and are also inalienable, unless converted pursuant to law into
alienable or disposable lands of the public domain. Historically, lands
reclaimed by the government are sui generis, not available for sale to private
parties unlike other alienable public lands. Reclaimed lands retain their
16 4S 2014-2015

San Beda College of Law CIVIL LAW REVIEW



inherent potential as areas for public use or public service. Alienable lands of
the public domain, increasingly becoming scarce natural resources, are to be
distributed equitably among our ever-growing population. To insure such
equitable distribution, the 1973 and 1987 Constitutions have barred private
corporations from acquiring any kind of alienable land of the public domain.
NOTES: DENR is vested with the power to authorize the reclamation of areas under water, while PEA is
vested with the power to undertake the physical reclamation of areas under water, whether directly or
through private contractors. DENR is also empowered to classify lands of the public domain into
alienable or disposable lands subject to the approval of the President. On the other hand, PEA is tasked
to develop, sell or lease the reclaimed alienable lands of the public domain.

Applicable CIVIL CODE Provisions: ART 420 and 422


Summary:
1. The 157.84 hectares of reclaimed lands comprising the Freedom Islands, now covered by
certificates of title in the name of PEA, are alienable lands of the public domain. PEA may lease these
lands to private corporations but may not sell or transfer ownership of these lands to private
corporations. PEA may only sell these lands to Philippine citizens, subject to the ownership limitations in
the 1987 Constitution and existing laws.
2. The 592.15 hectares of submerged areas of Manila Bay remain inalienable natural resources of
the public domain until classified as alienable or disposable lands open to disposition and declared no
longer needed for public service. The government can make such classification and declaration only
after PEA has reclaimed these submerged areas. Only then can these lands qualify as agricultural lands
of the public domain, which are the only natural resources the government can alienate. In their present
state, the 592.15 hectares of submerged areas are inalienable and outside the commerce of man.
3. Since the Amended JVA seeks to transfer to AMARI, a private corporation, ownership of 77.34
hectares of the Freedom Islands, such transfer is void for being contrary to Section 3, Article XII of the
1987 Constitution which prohibits private corporations from acquiring any kind of alienable land of the
public domain.
4. Since the Amended JVA also seeks to transfer to AMARI ownership of 290.156 hectares of still
submerged areas of Manila Bay, such transfer is void for being contrary to Section 2, Article XII of the
1987 Constitution which prohibits the alienation of natural resources other than agricultural lands of the
public domain. PEA may reclaim these submerged areas. Thereafter, the government can classify the
reclaimed lands as alienable or disposable, and further declare them no longer needed for public
service. Still, the transfer of such reclaimed alienable lands of the public domain to AMARI will be void in
view of Section 3, Article XII of the 1987 Constitution which prohibits private corporations from acquiring
any kind of alienable land of the public domain.

VILLARICO v. SARMIENTO
G.R. No. 136438, 11 November 2004
[SAYO]
FACTS:
Petitioner is the owner of a lot covered by TCT No. 95453. Petitioners lot is
separated from the Ninoy Aquino Avenue highway by a strip of land belonging
to the government. As the highway is elevated, the DPWH constructed
stairways at several portions of this strip of public land to enable the people to
have access to the highway. Respondents had a building constructed on a
portion of the said government land. Subsequently, by means of Deed of
Exchange of Real Property, petitioner acquired the same area of the
government land and was registered in petitioners name as TCT. No. 74430.
Petitioner filed a complaint for accion publiciana against respondents alleging
that respondents structures on the government land closed his right of way
to the Ninoy Aquino Avenue; and encroached on a portion of his lot covered
by TCT No. 74430. The RTC ruled that respondents have a better right of
17 4S 2014-2015

San Beda College of Law CIVIL LAW REVIEW



possession on the portion of the government land except for the portion
covered by TCT No. 74430. The CA affirmed in toto. Hence, this petition.
ISSUES:
1. Does petitioner have a better right over the portion of the government land
that closed petitioners right of way?
2. Is the petitioner entitled to possession over the portion that encroached the
land covered by TCT No. 74430?
RULING:
1. NO. It is not disputed that the lot on which petitioners alleged right of way
exists belongs to the state or property of public dominion. Public use is use
that is not confined to privileged individuals but is open to the indefinite
public. Considering that the lot on which the stairways were constructed is a
property of public dominion, it cannot be burdened by a voluntary easement of
right of way in favor of herein petitioner. In fact, its use by the public is mere
tolerance of the government through the DPWH. Petitioner cannot appropriate
it for himself. Verily, he cannot claim any right of possession over it. Neither
do the respondents have a better right of possession over the same.
2. YES. Being its owner, he is entitled to its possession.

OWNERSHIP: Rights Included in Ownership



JAVIER v. VERIDIANO II
G.R. No. L-48050, 10 October 1994
[SENTILLAS]
FACTS:
Sometime in January 1963 petitioner filed a Miscellaneous Sales Application
for Lot No. 1641;
Sometime in December 1970, alleging that she was forcibly dispossessed of a
portion of the land by a certain Ben Babol, petitioner instituted a complaint for
forcible entry before the City Court of Olongapo City;
On 7 November 1972 the City Court of Olongapo City dismissed the case on
the ground that "it appears to the Court that the Bureau of Lands has
considered the area in question to be outside Lot 1641 of the plaintiff. . . ."
and became final and executory on 30 April 1973;
Subsequently, on 17 December 1973, petitioner was granted Miscellaneous
Sales Patent No. 5548 and issued Original Certificate of Title No. P-3259
covering Lot No. 1641. Meanwhile, Ben Babol sold the property he was
occupying, including the portion of about 200 square meters in question, to a
certain Reino Rosete.

18 4S 2014-2015

San Beda College of Law CIVIL LAW REVIEW



On 29 June 1977, or after about four (4) years from the finality of the
dismissal of Civil Case No. 926, petitioner instituted a complaint for quieting of
title and recovery of possession with damages against Ben Babol and Reino
Rosete.
Instead of filing a responsive pleading, therein defendant Reino Rosete
(private respondent herein) moved to dismiss the complaint on the ground of
res judicata.
ISSUE: Is there identity of causes of action if the first case is an ejectment suit
and the second, an accion reivindicatoria?
Ruling:
NO. The first case is a complaint for forcible entry, where what is at Issue is
prior possession, regardless of who has lawful title over the disputed property.
The only Issue in an action for forcible entry is the physical or material
possession of real property, that is, possession de facto and not possession
de jure. A judgment rendered in a case for recovery of possession is
conclusive only on the question of possession and not on the ownership. It
does not in any way bind the title or affect the ownership of the land or
building.
On the other hand, the second case is in reality an action to recover a parcel
of land or an accion reivindicatoria under Art. 434 of the Civil Code, and
should be distinguished from the first case, which is an accion interdictal.
Accion reivindicatoria or accion de reivindicacion is an action whereby plaintiff
alleges ownership over a parcel of land and seeks recovery of its full
possession. It is different from accion interdictal or accion publiciana where
plaintiff merely alleges proof of a better right to possess without claim of title.
BUSTOS v. COURT OF APPEALS
G.R. No. 120784-85, January 24, 2001
[SINOCRUZ]
FACTS:
Paulino Fajardo died intestate leaving 4 children: Manuela, Trinidad, Beatriz
and Marcial. The heirs executed an extra-judicial partition of the estate of
Paulino. On the same day, Manuela sold her share (Lot 248) to Moses
Mendoza, Beatrizs husband, by a deed of sale.
Trinidad, however, was in physical possession of the land and refused to
surrender the same to Moses. Moses then filed a complaint for partition
claiming the share of Manuela which was sold to him. During the pendency
of hearing, Trinidad died and so her heirs executed an extra-judicial partition
of her estate which included Lot 248. After which, one of Trinidads heir sold
Lot 248 to spouses Viray.

19 4S 2014-2015

San Beda College of Law CIVIL LAW REVIEW



The trial court, nonetheless, rendered a decision in favor of Moses, granting
the partition and segregating the portion. By virtue of such decision, Moses
subsequently sold the Lot 248 to spouses Warlito and Herminia Bustos who
were lessees of Trinidads husband.
Since the spouses Bustos were in actual possession of the subject land, the
Virays, claiming to be rightful buyers Lot 248, filed with the MCTC an action
for unlawful detainer against them. The MCTC decided in favor of the Virays
and issued writs of execution and demolition against the spouses
Bustos. The spouses Bustos, however, filed a petition for certiorari,
prohibition and injunction which stayed the aforementioned writs.
ISSUE: Can petitioner-spouses Bustos be ejected from what is now their own
land?
Ruling:
NO. An owner who cannot exercise the seven (7) juses or attributes of
ownership- the right to possess, to use and enjoy, to abuse or consume, to
accessories, to dispose or alienate, to recover or vindicate and to the fruits is
a crippled owner.
In the unlawful detainer case, the Court of Appeals affirmed the decision of
the trial court as to possession on the ground that the decision has become
final and executory. Thus, petitioners may be evicted. However, in the accion
reinvidicatoria, the Court of Appeals affirmed the ownership of spouses
Bustos over the subject land. Hence, the court declared spouses Bustos as
the lawful owners of the land.
The stay of the execution in the unlawful detainer case is warranted by the
fact that spouses Bustos are now legal owners of the land. To eject them
from the land they owned would certainly result in injustice. Besides, the
issue of possession was rendered moot when the ownership was adjudicated
to the spouses Bustos by virtue of a deed of sale. Placing them in possession
is the necessary and logical consequence of such decision declaring them the
rightful owners.
HEIRS OF ROMAN SORIANO v. COURT OF APPEALS
G.R. No. 128177, 15 August 2001
[TILOS]
FACTS:
A piece of land located in Lingayen, Pangasinan is the disputed property in
this case. Said land was originally owned by one Adriano Soriano,
subsequently it was leased for a period of 15 years to the Spouses David and
Consuelo with RAMON SORIANO, son of Adriano and herein petitioner,
acting as caretaker/tenant of the property during the duration of the lease.
Upon the death of Adriano the lot he owned was divided into TWO and given
to his heirs. One of the lots inherited was sold to the Spouses ABALOS, here.
The other lot was also bought by the Spouses Abalos although not completely
20 4S 2014-2015

San Beda College of Law CIVIL LAW REVIEW



(only of the lot). The lots in question were subsequently registered in the
name of the Spouses Abalos. The courts later declared them to be the
undisputed owners thereof. Soriano questions their ownership of the land and
so filed cases against the spouses. Currently Soriano is still in possession of
the land claiming rights of Security of Tenure as a tenant of the land.
ISSUE: May a winning party in a land registration case effectively eject the
possessor thereof, whose security of tenure rights are still pending
determination before the DARAB?
RULING:
NO. Possession and ownership are distinct legal concepts. Possession is the
holding of a thing or the enjoyment of a right. Literally, to possess means to
actually and physically occupy a thing with or without right. A judgment of
ownership does not necessarily include possession as a necessary incident.
Such declaration pertains only to OWNERSHIP and does not automatically
include possession. This is especially true in the case at bar wherein
petitioner is occupying the land allegedly in the concept of an agricultural
tenant. The court says allegedly due to the fact that there is still a pending
case in the DARAB (Department of Agrarian Reform and Adjudication Board)
on the Issue. The Issue of ownership of the subject land has been laid to rest
by final judgment; however the right of possession is yet to be resolved. The
Tenancy Act, which protects the rights of agricultural tenants, may limit the
exercise of rights by the lawful owners. The exercise of the rights of
ownership yields to the exercise of the rights of an agricultural tenant. Since
the rights of Soriano to possess the land are still pending litigation in the
DARAB he is protected from dispossession of the land until final judgment of
said court unless Sorianos occupancy is found by the court to be unlawful.
DE GARCIA v. COURT OF APPEALS
G.R. No. L-20264, 30 January 1971
FACTS:
Angelina D. Guevarra seeks recovery from Consuelo de Garcia of one ladys
diamond ring 18cts.white gold mounting, with one 2.05 cts. diamond-solitaire,
and four brills 0.10 cts. total weight, which she bought on October 27, 1947
from R. Rebullida, Inc. Respondent Guevarras evidence tend to show that
while talking to petitioner Garcia, she recognized her ring on the finger of Mrs.
Garcia and inquired where she bought it. Guevarra explained that the ring
was stolen from her house. The ring fitted Guevarras finger. Upon
examination conducted by Mr. Rebullida with the aid of high power lens and
after consulting the stock card thereon, concluded that it was the very ring that
Guevarra bought from him in 1947. The ring was returned to Garcia who
despite a written request failed to deliver the ring to Guevarra. Later on, when
sheriff tried to serve the writ of seizure (replevin), Garcia refused to deliver the
ring which had been examined by Mr. Rebullida, claiming it was lost.

21 4S 2014-2015

San Beda College of Law CIVIL LAW REVIEW



ISSUES:
1. Can Respondent Angelina D. Guevarra recover the diamond ring which is
the subject of the dispute from petitioner Consuelo D. de Garcia?
2. Is Consuelo de Garcias alleged possession in good faith of the diamond
ring in question equivalent to title?
RULING:
1. YES. The controlling provision is Article 559 of the Civil Code. It
reads: The possession of movable property acquired in good faith is
equivalent to title. Nevertheless, one who has lost any movable or has been
unlawfully deprived thereof may recover it from the person in possession of
the same. If the possessor of a movable lost of which the owner has been
unlawfully deprived, has acquired it in good faith at a public sale, the owner
cannot obtain its return without reimbursing the price paid
therefor. Respondent Angelina D. Guevarra, having been unlawfully
deprived of the diamond ring in question, was entitled to recover it from
petitioner Consuelo S. de Garcia who was found in possession of the
same. The only exception the law allows is when there is acquisition in good
faith of the possessor at a public sale, in which case the owner cannot obtain
its return without reimbursing the price.
2. NO. Possession in good faith does not really amount to title, for the reason
that Article 1132 of the Civil Code provides for a period of acquisitive
prescription for movables through uninterrupted possession for four years in
good faith. The title established by the first clause of Article 559 is only a
presumptive title sufficient to serve as a basis for acquisitive prescription, that
the clause immediately following provides that one who has lost any movable
or has been unlawfully deprived thereof, may recover it from the person in
possession of the same.
RODIL ENTERPRISES, INC. v. COURT OF APPEALS, CARMEN BONDOC,
TERESITA BONDOC-ESTO, DIVISORIA FOOTWEAR and CHUA HUAY
SOON
G.R. No. 129609, 29 November 2001
[ABIERA]
FACTS:
Petitioner Rodil Enterprises (Rodil) is the lessee of the Ides ORacca building
(ORacca) since 1959. It was a former alien property over which the Republic
acquired ownership by virtue of RA 477. Rodil entered into a sublease
contract with respondents Bondoc, Bondoc-Esto, Divisoria Footware and
Chua Huay Soon, members of ORacca Building Tenants Association, Inc.
(Association). On January 8, 1987, Rodil offered to purchase the property
conformably with BP 233. On July 22, 1987, the Association also offered to
lease the same building through Department of General Services and Real
Estate Property Management (DGSREPM). Pending action on the offer of
Rodil to purchase the property, the Republic granted Rodils request for the
renewal of the lease contract on Sept. 23, 1987 for 5 more years. The renewal
contract however was disapproved by the DGSREPM Secretary De Jesus
22 4S 2014-2015

San Beda College of Law CIVIL LAW REVIEW



upon recommendation of DGSREPM Undersecretary Banas on the ground
that the offer of the Association was more beneficial to the Republic.
On October 1987, Rodil filed an action to enjoin the Association from
collecting rentals from the occupants or sub-lessees of ORacca. This was
granted by the trial court and upheld by the CA. On May 18, 1992, Rodil
signed a renewal contract for 10 more years of lease, which was approved by
the DENR Secretary. The Association then filed an action to set aside the
renewal contract, but the same was denied by the trial court.
Rodil then filed an action for unlawful detainer against herein respondents.
The MeTC upheld Rodils right to eject, which was then upheld by the RTC.
While the consolidated appeals were pending, the CA 2nd Division declared
the renewal contract between Rodil and the Republic as null and void. Rodil
moved for reconsideration but the same was denied which prompted it to file
an action for certiorari. The CA 4th Division likewise set aside the MeTC and
the RTCs decision and dismissed the action of Rodil for unlawful detainer.
ISSUES:
1. Did the Court of Appeals err in annulling petitioners renewal contract with
the Republic?
2. May petitioner validly eject herein respondents even though the former is
not in actual possession of the property?
RULING:
1. YES. The owner has the right to enjoy and dispose of a thing, without other
limitations than those established by law. Every owner has the freedom of
disposition over his property. It is an attribute of ownership, and this rule has
no exception. The Republic being the owner of the disputed property enjoys
the prerogative to enter into a lease contract with Rodil in the exercise of its
jus disponendi.
2. YES. In an action for unlawful detainer, the plaintiff need not have been in
prior physical possession. Respondents have admitted that they have not
entered into any lease contract with the Republic and that their continued
occupation of the subject property was merely by virtue of acquiescence.
Since the occupation of respondents was merely tolerated by the Republic,
the right of possession of the latter remained uninterrupted. It could therefore
alienate the same to anyone it choose. Unfortunately for respondents, the
Republic chose to alienate the subject premises to Rodil by virtue of a
contract of lease entered into on May 18, 1992. Resultantly, the petitioner had
the right to file the action for unlawful detainer against respondents as one
from whom possession of property has been unlawfully withheld.

23 4S 2014-2015

San Beda College of Law CIVIL LAW REVIEW



CORNELIO M. ISAGUIRRE v. FELICITAS DE LARA
G.R. No. 138053, 31 May 2000
[ALDUESO]
FACTS:
Alejandro de Lara was the original applicant-claimant for a Miscellaneous
Sales Application over a parcel of land filed with the Bureau of Lands. Upon
his death, he, was succeeded by his wife - respondent Felicitas de Lara, as
claimant. By virtue of a decision rendered by the Secretary of Agriculture and
Natural Resources, a subdivision survey was made and the area was reduced
to 1,000 square meters. On this lot stands a two-story residential-commercial
apartment declared for taxation purposes under TD 43927 in the name of
respondents sons - Apolonio and Rodolfo, both surnamed de Lara.
Felicitas obtained a loan from PNB. When she encountered financial
difficulties, respondent approached petitioner Cornelio M. Isaguirre. A
document denominated as "Deed of Sale and Special Cession of Rights and
Interests" was executed by respondent and petitioner, whereby the former
sold a 250 square meter portion of Lot No. 502, together with the two-story
commercial and residential structure standing thereon, in favor of petitioner,
for and in consideration of the sum of P5,000.
Subsequently, Apolonio and Rodolfo de Lara filed a complaint against
petitioner for recovery of ownership and possession of the two-story building
but was dismissed. Isaguirre filed a sales application over the subject property
on the basis of the deed of sale. His application was approved resulting in the
issuance of OCT No. P-11566. Meanwhile, the sales application of
respondent over the entire 1,000 square meters of subject property (including
the 250 square meter portion claimed by petitioner) was also given due
course, resulting in the issuance of OCT No. P-13038, in the name of
respondent.
Due to the overlapping of titles, petitioner filed an action for quieting of title
and damages with the RTC of Davao City against respondent. The RTC
rendered judgment in favor of petitioner declaring him to be the lawful owner
of the disputed property. However, the Court of Appeals reversed the trial
courts decision, holding that the transaction entered into by the parties was
an equitable mortgage, not a sale.
As a consequence of its decision, the appellate court also declared OCT
No.P-11566 issued in favor of petitioner to be null and void. In a case
docketed as G. R. No. 120832, this Court affirmed the decision of the Court of
Appeals and the petitioners MR was subsequently denied. Petitioner
contends that the CA has acted with grave abuse of discretion when it did not
held that the RTC erred in issuing a writ of possession as neither the CA
decision nor the SC decision mentioned, ordered immediate delivery of
possession to the respondent.

24 4S 2014-2015

San Beda College of Law CIVIL LAW REVIEW



ISSUES: Is the trial court correct in granting writ of possession against
petitioner when the court found that the respondent is the owner of said
property?
RULING:
YES. As the sole owner, respondent has the right to enjoy her property,
without any other limitations than those established by law.Corollary to such
right, respondent also has the right to exclude from the possession of her
property any other person to whom she has not transmitted such property.The
issuance of the writ of possession is but a necessary consequence of this
Court ruling in G.R. No 120832 affirming the validity of the original certificate
of title (OCT No. P-13038) in the name of respondent Felicitas de Lara, while
at the same time nullifying the original certificate of title (OCT No. P-11566) in
the name of petitioner Cornelio Isaguirre. Possession is an essential attribute
of ownership; thus, it would be redundant for respondent to go back to court
simply to establish her right to possess subject property. Contrary to
petitioners claims, the issuance of the writ of possession by the trial court did
not constitute an unwarranted modification of our decision in G.R. No.
120832, but rather, was a necessary complement thereto. It bears stressing
that a judgment is not confined to what appears upon the face of the decision,
but also those necessarily included therein or necessary thereto.
CUSTODIO v. COURT OF APPEALS
G.R. No. 116100, 9 February 1996
[ALON]
FACTS:
Pacifico Mabasa owns a parcel of land with a two-door apartment erected
thereon situated at Interior P. Burgos St., Palingon, Tipas, Taguig, Metro
Manila. Said property may be described to be surrounded by other immovable
(row of houses and a septic tank) pertaining to defendants Sps. Custodio and
Sps. Santos. There were two passageways available to be able to arrive at
Mabasas property that shall pass in between the previously mentioned row of
houses and septic tank. Sometime in February, 1982, one of the tenants of
Mabasa vacated the apartment and when plaintiff Mabasa went to see the
premises, he saw that there had been built an adobe fence in the first
passageway making it narrower in width; another passageway was entirely
enclosed upon the construction of an extended adobe fence. It was then that
the remaining tenants of said apartment vacated the area. Pacifico Mabasa,
thereafter, filed a civil case for the grant of an easement of right of way
against the defendants. The decision granted Pacifico Mabasa the right of
way with corresponding damages.
ISSUES: Was the award of damages properly awarded?
RULING:
NO. The mere fact that the plaintiff suffered losses does not give rise to a right
to recover damages. Whatever injury or damage may have been sustained by
Mabasa by reason of the rightful use of the said land by petitioners is
25 4S 2014-2015

San Beda College of Law CIVIL LAW REVIEW



damnum absque injuria. A person has a right to the natural use and
enjoyment of his own property, according to his pleasure, for all the purposes
to which such property is usually applied. As a general rule, therefore, there
is no cause of action for acts done by one person upon his own property in a
lawful and proper manner, although such acts incidentally cause damage or
an unavoidable loss to another, as such damage or loss is damnum absque
injuria. The act of petitioners in constructing a fence within their lot is a valid
exercise of their right as owners, hence not contrary to morals, good customs
or public policy. The law recognizes in the owner the right to enjoy and
dispose of a thing, without other limitations than those established by law.
PACENCIO ABEJARON, REPRESENTED BY HIS ATTORNEY-IN-FACT,
ALEJANDRO ABEJARON v. FELIX NABASA AND THE COURT OF
APPEALS
G.R. No. 84831, 20 June 2001
[BELARMINO]
FACTS:
Petitioner Pacencio Abejaron alleges that he is the actual and lawful
possessor and claimant of a 118 square meter portion of a 175 square meter
residential lot in Silway, General Santos City. Abejaron and his family started
occupying the 118 square meter land in 1945. They constructed a fence
around the area and build thereon their family home. At that time, the land
had not yet been surveyed. In 1949, petitioner improved their abode to
become a two-story house made of wood and nipa roofing. He also intoduced
improvements including a store, 5 coconut trees and avocado and banana
trees. Despite the improvement introduced on the said property, Felix
Nabasa, respondent, did not oppose or complain about the improvements.
Knowing that the land was public in character, he declared only his house for
taxation purposes.
Beginning 1955, Nabasa resided on the remaining 57 square meter portion of
the lot and built his house about 4 meters away from Abejarons house.
A survery by the Bureau of Land was thereafter conducted on the area in
controversy. Abejaron merely watched the survey and did not bother to apply
for title to the land on the belief that he could not secure title over it as it was
government property. Nabasa, however, without the knowledge and consent
of Abejaron, clandestinely, willfully, fraudulently, and unlawfully applied for
and caused the titling in his name of the entire 175 square meter lot.
Petitioner imputes bad faith on the part of Nabasa because he represented
himself to be the actual and lawful possessor of the entire lot despite
knowledge of Abejarons actual occupation and possession. On September
24, 1974, an Original Certificate of Title was issued to Nabasa. An action for
reconveyance was thereafter commenced by Abejaron against Nabasa.
ISSUE: Is it necessary, for an action for reconveyance to prosper, that the
petitioner be the absolute owner of the land in controversy?

26 4S 2014-2015

San Beda College of Law CIVIL LAW REVIEW



RULING:
YES. It is well-settled that reconveyance is a remedy granted only to the
owner of the property alleged to be erroneously titled in anothers name. For
an action for reconveyance based on fraud to prosper, it is essential for the
party seeking reconveyance to prove by clear and convincing evidence his
title to the property and the fact of fraud.
Petitioner does not claim to own the land in dispute. He in fact admits that he
believed the land in dispute was public in character, thus he did not declare it
for taxation purposes despite possession of it for a long time. Neither did he
apply for title over it on the mistaken belief that he could not apply for title of a
public land. Petitioners evidence does not constiture the well-nigh
incontrovertible evidence necessary to acquire title through possession and
occupation of the disputed land at least since January 24, 1947 as required by
Sec. 48(b) of the Public Land Act. The basic presumption is that lands of
whatever classification belong to the State and evidence of a land grant must
be well-nigh incontrovertible. As petitioner Abejaron has not adduced any
evidence of title to the land in controversy, whether by judicial confirmation of
title, or homestead, sale, or free patent, he cannot maintain an action for
reconveyance.

OWNERSHIP: Principle of Self-Help


GERMAN MANAGEMENT & SERVICES, INC. v. COURT OF APPEALS
G.R. No. 76217, 14 September 1989

FACTS:
The land subject of the case was formerly declared for taxation purposes in
the name of Sinforoso Mendoza prior to 1954 but is now declared in the name
of Margarito Mendoza. Petitioners are the daughters of Margarito Mendoza
while the respondent is the only daughter of Sinforoso Mendoza. Margarito
Mendoza and Sinforoso Mendoza were brothers, now deceased. During the
cadastral survey of the property on October 15, 1979 there was already a
dispute between Honorata M. Bolante and Miguel Mendoza, brother of
petitioners. Respondent was occupying the property in question.
ISSUE: Whether is or not the doctrine of self help applies in this case
RULING:
NO. The doctrine of self help, which the petitioners were using to justify their
actions, are not applicable in the case because it can only be exercised at the
time of actual or threatened dispossession which is absent in the case at bar.
Article 536 basically tells us that the owner or a person who has a better right
over the land must resort to judicial means to recover the property from
another person who possesses the land.

27 4S 2014-2015

San Beda College of Law CIVIL LAW REVIEW



CAISIP v. PEOPLE
G.R. No. L-28716, 18 November 1970
[BONAVENTE]
FACTS:
Gloria Cabalag is the wife of Marcelino Guevarra who cultivated a parcel of
land known as Lot 105-A of Hacienda Palico in Nasugbu, Batangas. Hacienda
Palico is owned by Roxas y Cia. The overseer of the said hacienda is Felix
Caisip. Roxas y Cia filed an action against Guevarra for forcible entry. The
Court ordered Guevarra to vacate the lot and to pay damages and accrued
rentals. A writ of execution was issued. Guevarra was given twenty days from
June 6, 1959 within which to leave the premises. On June 17, 1959, Gloria
Cabalag was seen weeding the portion of Lot 105-A which was a ricefield.
Caisip approached her and bade her to leave, but she refused to do so,
alleging that she and her husband had the right to stay there and that the
crops thereon belong to them. Caisip went to his co-defendants, Sgt. Rojales
and Cpl. Villadelrey, both of the local police, who were some distance away,
and brought them with him. Rojales told Gloria, who was then in a squatting
position, to stop weeding. As Gloria insisted on her right to stay in said lot,
Rojales grabbed her right hand and, twisting the same, wrested therefrom the
trowel she was holding. Thereupon, Villadelrey held her left hand and,
together with Rojales, forcibly dragged her northward towards a forested area,
where there was a banana plantation, as Caisip stood nearby, with a drawn
gun. Gloria shouted "Ina ko po! Ina ko po!" then her neighbors came and
asked the policemen why they were dragging her. The policemen having
answered that they would take Gloria to town which was on the west
Francisca Andino pleaded that Gloria be released, saying that, if their purpose
was as stated by them, she (Gloria) would willingly go with them. By this time,
Gloria had already been dragged about eight meters and her dress, as well as
her blouse were torn. She then agreed to proceed westward to the municipal
building.
ISSUE: Were the acts of the appellants justified under Article 429 of the New
Civil Code?
RULING:
NO. Article 429 of the Civil Code upon which appellants rely is obviously
inapplicable to the case at bar, for, having been given 20 days from June 6,
1959, within which to vacate Lot 105-A, complainant did not, within said
period, invade or usurp said lot. She had merely remained in possession
thereof, even though the hacienda owner may have become its co-possessor.
Appellants did not "repel or prevent an actual or threatened physical invasion
or usurpation." They expelled Gloria from a property of which she and her
husband were in possession and despite the fact that the Sheriff had explicitly
authorized them to stay in said property up to June 26, 1959. Appellants
herein had, by means of violence, and without legal authority therefor,
prevented the complainant from "doing something not prohibited by law,"
(weeding and being in Lot 105-A), and compelled her "to do something
against" her will (stopping the weeding and leaving said lot), "whether it be
28 4S 2014-2015

San Beda College of Law CIVIL LAW REVIEW



right or wrong," thereby taking the law into their hands, in violation of Art. 286
of the Revised Penal Code. Defendants Felix Caisip, Ignacio Rojales and
Federico Villadelrey, were convicted of the crime of Grave Coercion
aggravated by abuse of superior strength and disregard of the respect due the
offended party by reason of her sex.
PEOPLE v. PLETCHA, JR.
G.R. No. 19029, 27 June 1977
[CABBUAG]
FACTS:
Tito Pletcha is a farmer who owns a land which he has been cultivating for 19
years. A private corporation sought to take over the aforementioned land by
fencing the 4 hectares of his property. Such fencing was without authority or
court order. Because of this, Pletcha fought-off any the take over and resisted
the company. This forced the company to file a case for grave coercion
against Pletcha in the Municipal Court of Murcia, Negros Occidental.
Pletcha invokes the protective mantle of Article 429 of the Civil Code which
gives him the right to use reasonable force to exclude any person threatening
his exclusive ownership over the land.
The People asks for affirmance on the ground that the appellant should not
have taken the law into his own hands but rather have the courts decide the
case.
ISSUE: Can Pletcha properly invoked Article 429?
RULING:
YES. The principle of self-help authorizes the lawful possessor to use force
not only to prevent a threatened unlawful invasion or usurpation thereof; it is
sort of self-defense. It is lawful to repel force by force. He who merely uses
force to defend his possession does not possess by force. The use of such
necessary force to protect proprietary or possessory rights constitutes a
justifying circumstance under our penal laws.

OWNERSHIP: Use Injuring Rights of Third Persons


ANDAMO v. INTERMEDIATE APPELLATE COURT and MISSIONARIES
OF OUR LADY OF LA SALETTE, INC.
G.R. No. 74761, 6 November 1990
[CRON]
FACTS:
Petitioner spouses Andamo are the owners of a parcel of land situated in Biga
(Biluso) Silang, Cavite which is adjacent to that of private respondent,
Missionaries of Our Lady of La Salette, Inc., a religious corporation.
29 4S 2014-2015

San Beda College of Law CIVIL LAW REVIEW



Within the land of respondent corporation, waterpaths and contrivances,
including an artificial lake, were constructed, which allegedly inundated and
eroded petitioners' land, caused a young man to drown, damaged petitioners'
crops and plants, washed away costly fences, endangered the lives of
petitioners and their laborers during rainy and stormy seasons, and exposed
plants and other improvements to destruction.
Petitioners instituted a criminal action for destruction by means of inundation
under Article 324 of the Revised Penal Code and a civil action for damages
under Articles 2176 and 2177 of the Civil Code. The Court dismissed the civil
case in view of the pendency of the criminal action.
ISSUE: Can a corporation, which has built through its agents, waterpaths,
water conductors and contrivances within its land, thereby causing inundation
and damage to an adjacent land, be held civilly liable for damages under
Articles 2176 and 2177 of the Civil Code on quasi-delicts such that the
resulting civil case can proceed independently of the criminal case?
RULING:
YES. It must be stressed that the use of one's property is not without
limitations. Article 431 of the Civil Code provides that "the owner of a thing
cannot make use thereof in such a manner as to injure the rights of a third
person." SIC UTERE TUO UT ALIENUM NON LAEDAS. Moreover, adjoining
landowners have mutual and reciprocal duties which require that each must
use his own land in a reasonable manner so as not to infringe upon the rights
and interests of others. Although we recognize the right of an owner to build
structures on his land, such structures must be so constructed and maintained
using all reasonable care so that they cannot be dangerous to adjoining
landowners and can withstand the usual and expected forces of nature. If the
structures cause injury or damage to an adjoining landowner or a third person,
the latter can claim indemnification for the injury or damage suffered.
A quasi-delict or culpa aquiliana is a separate legal institution under the Civil
Code with a substantivity all its own, and individuality that is entirely apart and
independent from a delict or crime a distinction exists between the civil
liability arising from a crime and the responsibility for quasi-delicts or culpa
extra-contractual. The same negligence causing damages may produce civil
liability arising from a crime under the Penal Code, or create an action for
quasi-delicts or culpa extra-contractual under the Civil Code.

30 4S 2014-2015

San Beda College of Law CIVIL LAW REVIEW


OWNERSHIP: Surface Rights


ANDAMO v. INTERMEDIATE APPELLATE
COURT
G.R. No. 74761, 6 November 1990
[CAINDAY]

FACTS:
Sps. Andamo owned a parcel of land adjacent to the land of the Missionaries
of Our Lady of La Salette, Inc. Within the land of the latter, water paths and
contrivances, including an artificial lake were constructed, which allegedly
inundated and eroded petitioners land, caused a young man to drown,
damaged petitioners crops and plants, washed away costly fences,
endangered the lives of petitioners and their laborers and some other
destructions.
ISSUE: Did respondents improperly exercise their surface right?
RULING:
YES. Adjoining landowners have mutual and reciprocal duties which require
that each must use his own land in a reasonable manner so as not to infringe
upon the rights and interests of others. Although we recognize the right of an
owner to build structures on his land, such structures must be so constructed
and maintained using all reasonable care so that they cannot be dangerous to
adjoining landowners and can withstand the usual and expected forces of
nature. If the structures cause injury or damage to an adjoining landowner or
a third person, the latter can claim indemnification for the injury or damage
suffered.

OWNERSHIP: Accession Discreta


BACHRACH MOTOR CO., INC. v. TALISAY-SILAY CO., INC., ET AL.
G.R. No. 35223, 17 September 1931
[DIVINO]

FACTS:
On December 22, 1923, the Talisay-Silay Milling Co., Inc., was indebted to
the Philippine National Bank. To secure the payment of its debt, it succeeded
in inducing its planters, among whom was Mariano Lacson Ledesma, to
mortgage their land to the creditor bank. In order to compensate those
planters for the risk they were running with their property under the mortgage,
the aforesaid central, by a resolution passed on that same date, undertook to
credit the owners of the plantation thus mortgaged every year with a sum
equal to 2% of the debt secured according to yearly balance.
Later on, a complaint was filed by the Bachrach Motor Co., Inc., against the
Talisay-Silay Milling Co., Inc., for the delivery of the amount P13,850 as
bonus in favor of Mariano Lacson Ledesma. The Philippine National Bank
31 4S 2014-2015

San Beda College of Law CIVIL LAW REVIEW



filed a third party claim alleging a preferential right to receive any amount
which Mariano Lacson Ledesma might be entitled to from the Talisay-Silay
Milling Co. as bonus, because that would be civil fruits of the land mortgaged
to said bank by said debtor for the benefit of the central referred to. The trial
court held that the Bachrach Motor Co., Inc., had a preferred right to receive
the amount of P11,076.02 which was Mariano Lacson Ledesma's bonus, and
it ordered the defendant central to deliver said sum to the plaintiff. Hence, the
appeal of PNB.
ISSUE: Is the bonus in question (the compensation for the risk of having
subjected one's land to a lien in favor of the bank) considered civil fruits?
RULING:
NO. Article 355 of the Civil Code considers three things as civil fruits: First,
the rents of buildings; second, the proceeds from leases of lands; and, third,
the income from perpetual or life annuities, or other similar sources of
revenue. It is to be noted that the said bonus bears no immediate, but only a
remote accidental relation to the land mentioned, having been granted as
compensation for the risk of having subjected one's land to a lien in favor of
the bank, for the benefit of the entity granting said bonus. If this bonus be
income or civil fruits of anything, it is income arising from said risk, or, if one
chooses, from Mariano Lacson Ledesma's generosity in facing the danger for
the protection of the central, but certainly it is not civil fruits or income from the
mortgaged property, which, as far as this case is concerned, has nothing to
do with it.
EQUATORIAL REALTY DEVELOPMENT, INC. v. MAYFAIR THEATRE,
INC.,
G.R. NO. 133879, 21 November 2001
[DOMINGO]
FACTS:
[The case arose out of an earlier case decided by the Court on November 21,
1996.]
Carmelo & Bauermann, Inc. used to own a parcel of land, together with two 2storey buildings constructed thereon in Manila. On June 1, 1967, Carmelo
entered into a Contract of Lease with Mayfair Theater Inc. for a period of 20
years. The lease covered a portion of the second floor and mezzanine of a
two-storey building, wherein the Maxim Theatre was built. Two years later,
Mayfair entered into a second Contract of Lease with Carmelo for the lease of
another portion of the latter's property. In that space, Mayfair put up another
movie house known as Miramar Theater. The Contract of Lease was likewise
for a period of 20 years. Both leases contained a provision granting Mayfair a
right of first refusal to purchase the subject properties.
However, on July 30, 1978 within the 20-year-lease term the subject
properties were sold by Carmelo to Equatorial Realty Development, Inc. for
the total sum of P11,300,000, without their first being offered to Mayfair. As a
32 4S 2014-2015

San Beda College of Law CIVIL LAW REVIEW



result of the sale of the subject properties to Equatorial, Mayfair filed a
Complaint before the Regional Trial Court of Manila for the annulment of the
Deed of Absolute Sale between Carmelo and Equatorial, specific
performance, and damages. The trial court ruled in favor of Carmelo and
Equatorial. On appeal, the decision of the lower court was reversed. On a
Petition for Review filed in the SC, the Court rescinded the Deed of Absolute
Sale between Carmelo and Mayfair. Such decision became final and
executory in 1997. In April 1997, Mayfair filed a Motion for Execution, which
the trial court granted. However, Carmelo could no longer be located. Thus,
following the order of execution of the trial court, Mayfair deposited with the
clerk of court a quo its payment to Carmelo. The lower court issued a Deed of
Reconveyance in favor of Carmelo and a Deed of Sale in favor of Mayfair.
Meanwhile, in September 1997, Equatorial filed with the RTC of Manila, an
action for the collection of a sum of money against Mayfair, claiming payment
of rentals or reasonable compensation for the defendant's use of the subject
premises after its lease contracts had expired. In its Complaint, Equatorial
alleged among other things that the Lease Contract covering the premises
occupied by Maxim Theater expired on May 31, 1987, while the Lease
Contract covering the premises occupied by Miramar Theater lapsed on
March 31, 1989. Representing itself as the owner of the subject premises by
reason of the Contract of Sale on July 30, 1978, it claimed rentals arising from
Mayfair's occupation thereof.
ISSUE: Is Equatorial Realty entitled to payment of back rentals?
RULING:
NO. Rent is a civil fruit that belongs to the owner of the property producing
it by right of accession. Consequently and ordinarily, the rentals that fell due
from the time of the perfection of the sale to petitioner until its rescission by
final judgment should belong to the owner of the property during that period.
In the Law on Sales, ownership of the thing sold is a real right, which the
buyer acquires only upon delivery of the thing to him in any of the ways
specified in articles 1497 to 1501, or in any other manner signifying an
agreement that the possession is transferred from the vendor to the vendee.
Delivery may be either actual or constructive, but both forms of delivery
contemplate the absolute giving up of the control and custody of the property
on the part of the vendor, and the assumption of the same by the vendee. It is
an act by which one party parts with the title to and the possession of the
property, and the other acquires the right to and the possession of the same.
In the case, considering that the petitioner never took actual control and
possession of the property sold, in view of respondent's timely objection to the
sale and the continued actual possession of the property, it can be said that
respondent's opposition to the transfer of the property by way of sale to
Equatorial was a legally sufficient impediment that effectively prevented the
passing of the property into the latter's hands. The objection took the form of a
court action impugning the sale which was rescinded by a judgment rendered
by this Court in the mother case. Equatorial as buyer did not acquire a right to
33 4S 2014-2015

San Beda College of Law CIVIL LAW REVIEW



the fruits of the thing sold from the time the obligation to deliver the property to
petitioner arose.
Moreover, the Decision in the mother case stated that "Equatorial x x x has
received rents" from Mayfair "during all the years that this controversy has
been litigated." and the payment thereof of Mayfair to petitioner should not be
interpreted to mean either actual delivery or ipso facto recognition of
Equatorial's title. The rental payments made by Mayfair should not be
construed as a recognition of Equatorial as the new owner. They were made
merely to avoid imminent eviction.

OWNERSHIP: Accession Continua


DAMIAN IGNACIO ET AL v. ELIAS HILARIO, ET AL.
G.R. No. L-175, 30 April 1946
[ESTEBAN]

FACTS:
Elias Hilario and his wife Dionisia Dres filed a case before the CFI of
Pangasinan against Damian, Francisco and Luis, surnamed Ignacio,
concerning the ownership of a parcel of land, partly rice-land and partly
residential. After the trial of the case, the lower court, presided over by Hon.
Alfonso Felix, rendered judgment holding plaintiffs as the legal owners of the
whole property but conceding to defendants the ownership of the houses and
granaries built by them on the residential portion with the rights of a possessor
in good faith, in accordance with Article 361 of the Civil Code.
Subsequently, in a motion filed in the same Court of First Instance the
plaintiffs prayed for an order of execution alleging that since they chose
neither to pay defendants for the buildings nor to sell to them the residential
lot, said defendants should be ordered to remove the structure at their own
expense and to restore plaintiffs in the possession of said lot.
ISSUE: May an owner in good faith eject a builder in good faith without
choosing either to appropriate the building for himself after payment of its
value or to sell his land to the builder in good faith?
RULING:
NO. The owner of the building erected in good faith on a land owned by
another, is entitled to retain the possession of the land until he is paid the
value of his building, under Article 453. The owner of the land, upon the other
hand, has the option, under Article 361, either to pay for the building or to sell
his land to the owner of the building. But he cannot, as respondents here did,
refuse both to pay for the building and to sell the land and compel the owner
of the building to remove it from the land where it is erected. He is entitled to
such remotion only when, after having chosen to sell his land, the other party
fails to pay for the same.
34 4S 2014-2015

San Beda College of Law CIVIL LAW REVIEW



FLORENCIO IGNAO v. HON. INTERMEDIATE APPELLATE COURT, JUAN
IGNAO, substituted by his Legal Heirs, and ISIDRO IGNAO
G.R. No. 72876, 18 January 1991
[FERNANDEZ]
FACTS:
Petitioner Florencio Ignao and his uncles private respondents Juan Ignao and
Isidro Ignao were co-owners of a parcel of land with an area of 534 square
meters situated in Barrio Tabon, Kawit, Cavite. Pursuant to an action for
partition, the then CFI of Cavite directed the partition of the aforesaid land,
alloting 133.5 square meters to private respondents Juan and Isidro, and
giving the remaining portion with a total area of 266.5 square meters to
petitioner Florencio. However, no actual partition was ever effected.
Petitioner instituted a complaint for recovery of possession of real property
against private respondents alleging that the area occupied by the two (2)
houses built by private respondents exceeded the 133.5 square meters
previously alloted to them. The lower court found that the houses of Juan and
Isidro actually encroached upon a portion of the land belonging to Florencio.
The CFI ruled that although private respondents occupied a portion of
Florencio's property, they should be considered builders in good faith and
pursuant to Article 448 of the Civil Code, the owner of the land (Florencio)
should have the choice to either appropriate that part of the house standing
on his land after payment of indemnity or oblige the builders in good faith
(Juan and Isidro) to pay the price of the land.
However, the trial court observed that based on the facts of the case, it would
be useless and unsuitable for Florencio to exercise the first option since this
would render the entire houses of Juan and Isidro worthless. It ordered the
plaintiff Florencio Ignao to sell to the defendants Juan and Isidro Ignao that
portion of his property with an area of 101 square meters at P40.00 per
square meter, on which part the defendants had built their houses. On appeal,
the IAC affirmed the CFIs decision.
ISSUES:
1. Should the provisions of Article 448 apply to a builder in good faith on a
property held in common?
2. May the court choose from the options provided in Article 448 for the
petitioner?
RULING:
1. YES. When the co-ownership is terminated by a partition and it appears
that the house of an erstwhile co-owner has encroached upon a portion
pertaining to another co-owner which was however made in good faith, then
the provisions of Article 448 should apply to determine the respective rights of
the parties.

35 4S 2014-2015

San Beda College of Law CIVIL LAW REVIEW



2. NO. In Quemuel v. Olaes, the Court categorically ruled that the right to
appropriate the works or improvements or to oblige the builder to pay the
price of the land belongs to the landowner.
FILIPINAS COLLEGES, INC. v. MARIA GARCIA TIMBANG, ET AL.
G.R. No. L-12812, 29 September 1959
[J.FERNANDO]
FACTS:
The parties in this case are the spouses Timbang, the Filipinas Colleges, Inc.,
and Maria Gervacio Blas. Blas was declared to be a builder in good faith for
the school building constructed on the subject lot and entitled to be paid the
amount of the building. While Filipinas Colleges, Inc. was declared to have
acquired the rights of the spouses Timbang in the lot subject to the payment
of the value of the land. In case Filipinas Colleges, Inc. failed to deposit the
value of the land within the time fixed by the court, Filipinas Colleges would
lose all its rights to the land and the spouses Timbang would then become the
owners thereof. In that eventuality, the Timbangs would make known to the
court their option under Art. 448 of the Civil Code whether they would
appropriate the building in question, in which even they would have to pay
Filipinas Colleges, Inc. the sum of P19,000.00, or would compel the latter to
acquire the land and pay the price thereof.
It is contended by the appellants that because the builder in good faith has
failed to pay the price of the land after the owners thereof exercised their
option under Article 448 of the Civil Code, the builder lost his right of retention
provided in Article 546 and by operation of Article 445, the appellants as
owners of the land automatically became the owners ipso facto, the execution
sale of the house in their favor was superfluous.
ISSUE:
Is the failure of the builder to pay the value of the land, when such is
demanded by the land-owner, the latter becomes automatically the owner of
the improvement?
RULING:
NO. Under the terms of articles 448 and 546, it is true that the owner of the
land has the right to choose between appropriating the building by
reimbursing the builder of the value thereof or compelling the builder in good
faith to pay for his land. Even this second right cannot be exercised if the
value of the land is considerably more than that of the building. In addition to
the right of the builder to be paid the value of his improvement, Article 546
gives him the corollary right of retention of the property until he is indemnified
by the owner of the land. There is nothing in the language of these two article,
448 and 546, which would justify the conclusion of appellants that, upon the
failure of the builder to pay the value of the land, when such is demanded by
the land-owner, the latter becomes automatically the owner of the
improvement under Article 445.
36 4S 2014-2015

San Beda College of Law CIVIL LAW REVIEW



MANOTOK REALTY INC. v. TECSON
G.R. No. L-47475, 19 August 1988
[L. FERNANDO]
FACTS:
In a complaint filed by the petitioner Manotok Realty for recovery of
possession of land against private respondent Nilo Madlangawa, CFI ruled
declaring the said respondent a builder in good faith. CA affirmed and SC
dismissed for lack of merit. Petitioner then filed with the trial court motion for
the approval of the petitioner's exercise of option to appropriate the
improvements introduced by the private respondent on the property under
Article 448 of the Civil Code, and thereafter, for satisfaction of judgment(that
is final and executory) by ordering respondents to deliver possession of the
property in question to the petitioner. The motion was dismissed. Hence this
petition for mandamus alleging that the respondent judge committed grave
abuse of discretion in denying petitioners motion on the grounds that under
Articles 448 and 546 of the Civil Code, the exercise of option belongs to the
owner of the property, who is the petitioner herein, and that upon finality of
judgment, the prevailing party is entitled, as a matter of right, to its execution
which is only a ministerial act on the part of the respondent judge.Petitioner
also claimed that since the house of the private respondent (which was the
improvement built on the land owned by petitioner) was gutted by fire, the
execution of the decision would now involve the delivery of possession of the
disputed area by the private respondent to the petitioner.
ISSUE: Should the court Issue the writ of execution for the delivery of
possession by the builder in good faith of the subject land to the owner thereof
after a fire gutted the improvements built by the former over the said land
which serves as basis of his right to retain the same?
RULING:
YES. Where the improvements have been destroyed by a fortuitous event
without the fault of the landowner, the basis for the builders right to retain the
premises is extinguished; hence there is no other recourse for him but to
vacate the premises and deliver the same to the landowner.
Moreover, even without the fire, the answer will still be the same. The option
given by law under Article 448 of the Civil Code, i.e., either to retain the
premises and pay for the improvements thereon or to sell the said premises to
the builder in good faith belongs to the owner of the property. The only right
given to the builder in good faith is the right to reimbursement for the
improvements; the builder cannot compel the owner of the land to sell such
land to the former.
THEREFORE, the judge cannot deny the issuance of a writ of execution for
the delivery of possession of the subject land, had the owner chose to retain
the premises and pay for the improvements thereon, because the other party
was adjudged a builder in good faith or on the ground of peculiar
circumstances which supervened after the institution of this case.
37 4S 2014-2015

San Beda College of Law CIVIL LAW REVIEW



BERNARDO v. BATACLAN
G.R. No. L-44606, 28 November 1938
[FLORES]
FACTS:
Plaintiff Vicente Bernardo acquired a parcel of land from Pastor Samonte
through a contract of sale. Thereafter, Bernardo instituted a case against said
Vendor to secure a possession of the land. Bernardo was able to obtain a
favorable decision from the court. He then found defendant Catalino Bataclan
in the said premises. It appears that he has been authorized by the former
owners, as far back as 1922, to clear the land and make improvements
thereon. Thus, plaintiff instituted a case against Bataclan in the Court of First
Instance of Cavite. In this case, plaintiff was declared the owner of the land
but the defendant was held to be a possessor in good faith, entitled to
reimbursement in the total sum of P1,646, for work done and improvements
made.
On appeal, the court made some modifications by allowing the defendant to
recover compensation amounting to P2,212 and by reducing the price at
which the plaintiff could require the defendant to purchase the land in question
from P300 down to P200 per hectare.
Plaintiff conveyed to the court his desire to require the defendant to pay the
land at the rate of P200 per hectare or a total price of P18,000 for the whole
tract of land. Defendant failed to pay the land.
ISSUE: Can Bataclan still exercise his right of retention?
RULING:
NO. The Civil Code confirms time-honored principles of the law of
property. One of these is the principle of accession whereby the owner of
property acquires not only that which it produces but that which is united to it
either naturally or artificially (Article 353). Whatever is built, planted or sown
on the land of another, and the improvements on repairs made thereon,
belong to the owner of the land (Article 358). Where, however, the planter,
builder, or sower has acted in good faith, a conflict of rights arises between
the owners and it becomes necessary to protect the owner of the
improvements without causing injustice to the owner of the land. In view of
the impracticability of creating what Manresa calls a state of forced
ownership, the law has provided a just and equitable solution by giving the
owner of the land the option to acquire the improvements after payment of the
proper indemnity or to oblige the builder or planter to pay for the land and the
sower to pay the proper rent (Article 361). It is the owner of the land who is
allowed to exercise the option because his right is older and because, by the
principle of accession, he is entitled to ownership of the accessory thing. The
law requires no more than that the owner of the land should choose between
indemnifying the owner of the improvements or requiring the latter to pay for
the land. When defendant Bataclan failed to pay the land, he herein lost his
right of retention.
38 4S 2014-2015

San Beda College of Law CIVIL LAW REVIEW



HEIRS OF RAMON DURANO, SR. v. SPOUSES UY
G.R. No. 136456, 24 October 2000
[JAVIER]
FACTS:
Petitioners instituted an action for damages against the respondents because
of allegedly officiating a hate campaign against them over petitioners socalled invasion of respondents alleged properties.
Even before many of the respondents received notices to vacate, men who
identified themselves as employees of Durano & Co. proceeded to bulldoze
the lands occupied by various respondents, destroying in their wake the
plantings and improvements made by the respondents therein.
In their counterclaim, respondents alleged that petitioners acts deprived most
of them of their independent source of income and have made destitutes of
some of them. Also, petitioners have done serious violence to respondents
spirit, as citizens and human beings, to the extent that one of them had been
widowed by the emotional shock that the damage and dispossession has
caused. Thus, in addition to the dismissal of the complaint, respondents
demanded actual damages for the cost of the improvements they made on
the land, together with the damage arising from the dispossession itself; moral
damages for the anguish they underwent as a result of the high-handed
display of power by petitioners in depriving them of their possession and
property; as well as exemplary damages, attorneys fees and expenses of
litigation.
The court found that the properties properly belonged to the respondents by
way of purchase or inheritance and that the titles procured in the name of
Ramon Durano III were attended by fraud.
ISSUE: What is the basis of damages to be paid by a builder in bad faith under
Art. 451 of the Civil Code?

RULING:
The right of the owner of the land to recover damages from a builder in bad faith
is clearly provided for in Article 451 of the Civil Code. Although said Article 451
does not elaborate on the basis for damages, the Court perceives that it should
reasonably correspond with the value of the properties lost or destroyed as a
result of the occupation in bad faith, as well as the fruits (natural, industrial or
civil) from those properties that the owner of the land reasonably expected to
obtain.

The owner of the land has three alternative rights: (1) to appropriate what has
been built without any obligation to pay indemnity therefor, or (2) to demand
that the builder remove what he had built, or (3) to compel the builder to pay
the value of the land. In any case, the landowner is entitled to damages under
Article 451.
39 4S 2014-2015

San Beda College of Law CIVIL LAW REVIEW



BALLATAN v. COURT OF APPEALS
G.R. No. 125683, 2 March 1999
[KING]
FACTS:
The instant case arose from a dispute over forty-two (42) square meters of
residential land belonging to petitioners.
In 1985, petitioner Ballatan constructed her house on Lot No. 24. During the
construction, she noticed that the concrete fence and side pathway of the
adjoining house of respondent Winston Go encroached on the entire length of
the eastern side of her property. Her building contractor informed her that the
area of her lot was actually less than that described in the title. Forthwith,
Ballatan informed respondent Go of this discrepancy and his encroachment
on her property. Respondent Go, however, claimed that his house, including
its fence and pathway, were built within the parameters of his father's lot; and
that this lot was surveyed by Engineer Jose Quedding, the authorized
surveyor of the Araneta Institute of Agriculture (AIA), the owner-developer of
the subdivision project.
Petitioner Ballatan called the attention of the AIA to the discrepancy of the
land area in her title and the actual land area received from them. The AIA
authorized another survey of the land by Engineer Jose N. Quedding.
In a report dated February 28, 1985, Engineer Quedding found that the lot
area of petitioner Ballatan was less by a few meters and that of respondent Li
Ching Yao, which was three lots away, increased by two (2) meters. In short,
Lots Nos. 25, 26 and 27 moved westward to the eastern boundary of Lot No.
24.
ISSUES:
1. Are respondents builders in good faith?
2. What are the rights of petitioners on said dispute?
RULING:
1. YES. The appellate court, however, found that it was the erroneous survey
by Engineer Quedding that triggered these discrepancies. Respondents Go
are deemed builders in good faith until the time petitioner Ballatan informed
them of their encroachment on her property.
Furthermore, there is no evidence, much less, any allegation that respondent
Li Ching Yao was aware that when he built his house he knew that a portion
thereof encroached on respondents Go's adjoining land.
Good faith is always presumed, and upon him who alleges bad faith on the
part of a possessor rests the burden of proof. All the parties are presumed to
have acted in good faith.

40 4S 2014-2015

San Beda College of Law CIVIL LAW REVIEW



2. Petitioners, as owners of Lot No. 24, may choose to purchase the
improvement made by respondents Go on their land, or sell to respondents
Go the subject portion. If buying the improvement is impractical as it may
render the Go's house useless, then petitioners may sell to respondents Go
that portion of Lot No. 24 on which their improvement stands. If the Go's are
unwilling or unable to buy the lot, then they must vacate the land and, until
they vacate, they must pay rent to petitioners. Petitioners, however, cannot
compel respondents Go to buy the land if its value is considerably more than
the portion of their house constructed thereon. If the value of the land is much
more than the Go's improvement, then respondents Go must pay reasonable
rent. If they do not agree on the terms of the lease, then they may go to court
to fix the same.
In the event that petitioners elect to sell to respondents Go the subject portion
of their lot, the price must be fixed at the prevailing market value at the time of
payment.
SPOUSES DEL CAMPO v. ABESIA
G.R. No.L-49219, 15 April 1988
[LIBOON]
FACTS:
An action for partition was filed by plaintiffs in the CFI of Cebu. Plaintiffs and
defendants are co-owners pro indiviso of this lot in the proportion of and 1/3
share each, respectively. The trial court appointed a commissioner who
recommended that the lot be divided into two portion with the area of 30 sqm
for the plaintiffs and 15 sqm for the defendants. The houses of plaintiffs and
defendants were surveyed and shown on the sketch plan. The house of
defendants occupied the portion with an area of 5 square meters of Lot 1161A of plaintiffs. The parties manifested their conformity to the report and asked
the trial court to finally settle and adjudicate who among the parties should
take possession of the 5 square meters of the land in question.
ISSUE: Is Art 448 of the New Civil Code applicable in cases where the
disputed land is co-owned?
RULING:
NO. Article 448 of the New Civil Code provides as follows:
Art. 448. The owner of the land on which anything has been built, sown, or planted in good
faith, shall have the right to appropriate as his own the works, sowing or planting, after
payment of the indemnity provided for in articles 546 and 548, or to oblige the one who built
or planted to pay the price of the land, and the one who sowed, the proper rent. However, the
builder or planter cannot be obliged to buy the land if its value is considerably more than that
of the building or trees. In such case, he shall pay reasonable rent, if the owner of the land
does not choose to appropriate the building or trees after proper indemnity. The parties shall
agree upon the terms of the lease and in case of disagreement, the court shall fix the terms
thereof.

The court a quo correctly held that Article 448 of the Civil Code cannot apply
where a co-owner builds, plants or sows on the land owned in common for
41 4S 2014-2015

San Beda College of Law CIVIL LAW REVIEW



then he did not build, plant or sow upon land that exclusively belongs to
another but of which he is a co-owner. The co-owner is not a third person
under the circumstances, and the situation is governed by the rules of coownership.
However, when, as in this case, the co-ownership is terminated by the
partition and it appears that the house of defendants overlaps or occupies a
portion of 5 square meters of the land pertaining to plaintiffs which the
defendants obviously built in good faith, then the provisions of Article 448 of
the new Civil Code should apply. Manresa and Navarro Amandi agree that the
said provision of the Civil Code may apply even when there was co-ownership
if good faith has been established.
PACIFIC FORMS, INC. v. ESGUERRA
G.R. No. L-21783, 29 November 1969
[LIM]
FACTS:
Defendant appellant, Carried Lumber Co. delivered and sold construction
materials to Insular Farms, Inc. such was utilized for construction of six
buildings at its compound in Bolinao Pangasinan within the period of
Oct.1,1956- Mar.2, 1957.
The total procurement amounted to P15, 000.00 but Insular Farms, Inc paid
only partial amount of P4, 710.18. Carried Lumber, Co instituted a civil case to
recover the unpaid balance before the CFI of Pangasinan which sustained the
companys claim, writ of execution was issued. The sheriff levied the six
buildings.
The plaintiff- appellee, Pacific Farms, Inc, filed a third party complaint
asserting absolute ownership of the buildings it was sold by Insular Farms
seven prior to the civil case. They seek to nullify the levy and judicial sale with
damages. The corporation contended to be innocent buyer in good faith for
value.
Hence, this appeal.
ISSUE: Is there a materialman's lien over the six buildings in favor of the
appellant to be paid by the buyer of property?
RULING:
YES. Therefore, applying article 447 by analogy, we perforce consider the
buildings as the principal and the lumber and construction materials that went
into their construction as the accessory. Thus the appellee, if it does own the
six buildings, must bear the obligation to pay for the value of the said
materials; the appellant which apparently has no desire to remove the
materials, and, even if it were minded to do so, cannot remove them without
necessarily damaging the buildings has the corresponding right to recover
the value of the unpaid lumber and construction materials.
42 4S 2014-2015

San Beda College of Law CIVIL LAW REVIEW



The court noted evident circumstances that negate the appellee's claim of
being innocent buyer in good faith for value:

In the deed of absolute sale, exhibit 1, the Insular Farms, Inc. (vendor) was represented in
the contract by its president, J. Antonio Araneta. The latter was a director of the appellee (Pacific
Farms, Inc.) and was the counsel who signed the complaint filed by the appellee in the court below.
J. Antonio Araneta was, therefore, not only the president of the Insular Farms, Inc. but also a
director and counsel of the appellee.

During the trial of civil case the Insular Farms, Inc. was represented by Attorney Amado
Santiago, Jr. of the law firm of J. Antonio Araneta. The latter was one of the counsels of the Pacific
Farms, Inc.

PECSON v. COURT OF APPEALS


G.R. No. 115814, 26 May 1995
[MADRILENO]
FACTS:
Petitioner Pedro P. Pecson was the owner of a commercial lot located in
Kamias Street, Quezon City, on which he built a four-door two-storey
apartment building. For his failure to pay realty taxes amounting to twelve
thousand pesos (P12,000.00), the lot was sold at public auction by the city
Treasurer of Quezon City to Mamerto Nepomuceno who in turn sold it on 12
October 1983 to the private respondents, the spouses Juan Nuguid and
Erlinda Tan-Nuguid, for one hundred three thousand pesos (P103,000.00).
The petitioner challenged the validity of the auction sale in Civil Case No. Q41470 before the RTC of Quezon City. In its decision of 8 February 1989, the
RTC dismissed the complaint, but as to the private respondents' claim that the
sale included the apartment building, it held that the issue concerning it was
"not a subject of the litigation." In resolving the private respondents' motion to
reconsider this issue, the trial court held that there was no legal basis for the
contention that the apartment building was included in the sale.
After an entry of judgment was made, the Sps. Nuguid filed a motion with the
RTC for a motion for delivery of possession of the lot and the apartment bldg
citing Art. 546 of the CC. The RTC issued an order declaring that the owner of
the lot and apartment bldg were the Sps. Nuguid and to pay the construction
cost of the apartment before a writ of possession would be issued and to pay
rent to the spouses. Pecson moved for reconsideration but the Trial court did
not act on it, instead it issued a writ of possession. The CA affirmed in part the
decision declaring the cost of construction can be offset from the amount of
rents to be collected and that since Sps. Nuguid opted to appropriate the
improvement, Pecson is entitled to be reimbursed the cost of construction at
the time it was built in 1965 which is at P53k and the right the retain the
improvement until full indemnity is paid.
ISSUE: Does Art. 448 and 546 apply in the case at bar
RULING:
YES. With regard to Art. 448, the provision on indemnity may be applied in
analogy. Whoever is the owner of the land may appropriate whatever has
43 4S 2014-2015

San Beda College of Law CIVIL LAW REVIEW



been built, planted or sown after paying indemnity. However, it does not apply
when the owner of the land is also the builder of the works on his own land
who later on loses ownership by sale or donation.
On the other hand, Art. 546 refers to the necessary and useful expenses
which shall be refunded to the possessor in good faith with right of retention.
However, it does not state how to determine the value of the useful
improvement. The respondent court and private respondents alike claims that
the construction cost in 1965 constitute sufficient reimbursement, however,
this is contrary to previous rulings which declares that the value to the
reimbursed should be the present market value of said improvements so as
not to unjustly enrich either of the parties. The trial court erred in ordering
Pecson to pay rent since the Sps. Nuguid has yet to pay the indemnity,
therefore Pecson has the right to retain the improvements and the income
thereof. The case was remanded to the trial court for determination of the
current market value of the apartment bldg and ordered the Sps. to pay
Pecson otherwise it shall be restored to Pecson until payment of indemnity.

OWNERSHIP: Good Faith


TECHNOGAS PHILIPPINES MANUFACTURING
COURT OF APPEALS AND EDUARDO UY
G.R. No. 108894, 10 February 1997
[MAGULTA]

CORPORATION

v.

FACTS:
Petitioner, Technogas Philippines Manufacturing Corporation (Technogas) is
the registered owner of a parcel of land in Paranaque, Metro Manila. The said
land was purchased by plaintiff from Pariz Industries Inc. in 1970, together
with all the buildings and improvements including the wall existing thereon. On
the other hand, respondent Eduardo Uy is the registered owner of a land
adjoining that of petitioner.
It was discovered in a survey that portions of the buildings and wall bought by
plaintiff together with the land from Pariz Industries are occupying a portion of
defendants adjoining land. Upon learning of the enroachment, plaintiff offered
to buy from defendant that particular portion occupied by portions of its
buildings and wall. Defendant, however, refused. In 1973, the parties entered
into a private agreement wherein plaintiff agreed to demolish the wall, thus
giving the defendant possession of a portion of his land previously enclosed
by plaintiffs wall. Uy later files a complaint before the Municipal Engineer of
Paranaque as well as before the Office of the Provincial Fiscal against
Technogas but the complaint did not prosper. Uy thereafter dug or caused to
be dug a cannal along Techogas wall, a portion of which collapsed. This led
to the filing by the petitioner of a supplemental complaint in the above
mentioned case and a separate criminal complaint for malicious mischief
against Uy and his wife. While trial of the case was in progress, plaintiff filed in
44 4S 2014-2015

San Beda College of Law CIVIL LAW REVIEW



Court a formal proposal for settlement of the case but said proposal, however,
was ignored by defendant.
The RTC ruled in favor of Technogas ordering Uy to sell to Technogas the
portion of land enroached by the building. The CA reversed the decision of the
RTC ruling that petitioner is a builder in bad faith because he is presumed to
know the metes and bounds of his property.
ISSUE: Is bad faith imputable to a registered owner of a land when a part of his
building enroached upon a neighbors land, simply because he is supposedly
presumed to know the boundaries of his land as described in his certificate of
title
RULING:
NO. Unless one is versed in the science of surveying, no one can determine
the precise extent or location of his property by merely examining his paper
title. Article 527 of the Civil Code presumes good faith, and since no proof
exists to show that the encroachment over a narrow, needle-shaped portion of
private respondent's land was done in bad faith by the builder of the
encroaching structures, the latter should be presumed to have built them in
good faith. It is presumed that possession continues to be enjoyed in the
same character in which it was acquired, until the contrary is proved. Good
faith consists in the belief of the builder that the land he is building on is his,
and his ignorance of any defect or flaw in his title. Further, "(w)here one
derives title to property from another, the act, declaration, or omission of the
latter, while holding the title, in relation to the property, is evidence against the
former. And possession acquired in good faith does not lose this character
except in case and from the moment facts exist which show that the
possessor is not unaware that he possesses the thing improperly or
wrongfully. The good faith ceases from the moment defects in the title are
made known to the possessor, by extraneous evidence or by suit for recovery
of the property by the true owner.
As to the question whether the same benefit can be invoked by petitioner is
not the builder of the offending structures bus possess them as buyer, the
court answered in the affirmative. There is no sufficient showing that petitioner
was aware of the encroachment at the time it acquired the property from Pariz
Industries. In any case, contrary proof has not overthrown the presumption of
good faith under Article 527 of the Civil Code, as already stated, taken
together with the disputable presumptions of the law on evidence. In the
second place, upon delivery of the property by Pariz Industries, as seller, to
the petitioner, as buyer, the latter acquired ownership of the property.
Consequently, petitioner is deemed to have stepped into the shoes of the
seller in regard to all rights of ownership over the immovable sold, including
the right to compel the private respondent to exercise either of the two options
provided under Article 448 of the Civil Code.

45 4S 2014-2015

San Beda College of Law CIVIL LAW REVIEW



PLEASANTVILLE DEVELOPMENT
APPEALS
G.R. No. 79688, 1 February 1996
[MANRIQUE]

CORPORATION

v.

COURT

OF

FACTS:
Edith Robillo purchased from petitioner a parcel of land designated as Lot 9 in
Pleasantville Subdivision. In 1975, respondent Eldred Jardinico bought the
rights to the lot from Robillo. At that time, Lot 9 was vacant. Upon completing
all payments, Jardinico secured from the Register of Deeds a Transfer
Certificate of Title in his name. It was then that he discovered that
improvements had been introduced on Lot 9 by respondent Wilson Kee, who
had taken possession thereof. It appears that on March 26, 1974, Kee bought
on installment Lot 8 of the same subdivision from C.T. Torres Enterprises, Inc.
(CTTEI), the exclusive real estate agent of petitioner. Under the Contract to
Sell on Installment, Kee could possess the lot even before the completion of
all installment payments. On January 20, 1975, Kee paid CTTEI the
relocation fee of P50.00 and another P50.00 on January 27, 1975, for the
preparation of the lot plan. These amounts were paid prior to Kees taking
actual possession of Lot 8. After the preparation of the lot plan and a copy
thereof given to Kee, CTTEI through its employee, Zenaida Octaviano,
accompanied Kees wife, Donabelle Kee, to inspect Lot 8. Unfortunately, the
parcel of land pointed by Octaviano was Lot 9. Thereafter, Kee proceeded to
construct his residence, a store, an auto repair shop and other improvements
on the lot. After discovering that Lot 9 was occupied by Kee, Jardinico
confronted him. The parties tried to reach an amicable settlement, but failed.
On January 30, 1981, Jardinicos lawyer wrote Kee, demanding that the latter
remove all improvements and vacate Lot 9. When Kee refused to vacate,
Jardinico filed a complaint for ejectment with damages against Kee. Kee, in
turn, filed a third-party complaint against petitioner and CTTEI.
ISSUE: Was the respondent a builder in good faith?
RULING:
YES. Good faith consists in the belief of the builder that the land he is building
on is his and his ignorance of any defect or flaw in his title. And as good faith
is presumed, petitioner has the burden of proving bad faith on the part of the
respondent (Kee). At the time he built improvements on Lot 8, Kee believed
that said lot was what he bought from petitioner. He was not aware that the lot
delivered to him was not Lot 8. Thus, Kee's good faith. Petitioner failed to
prove otherwise.

46 4S 2014-2015

San Beda College of Law CIVIL LAW REVIEW



GEMINIANO, ET. AL. v. COURT OF APPEALS
G.R. No. 120303, 24 July 1996
[MATEO]
FACTS:
Paulina Amado vda. de Geminiano, petitioners mother, owns a with an area
of 314 square meters. On a 12-square-meter portion of that lot stood the
petitioners' unfinished bungalow, which the petitioners sold in November 1978
to the private respondents for the sum of P6,000.00, with an alleged promise
to sell to the latter that portion of the lot occupied by the
house. Subsequently, the petitioners' mother executed a contract of lease
over a 126 square-meter portion of the lot, including that portion on which the
house stood, in favor of the private respondents for P40.00 per month for a
period of seven years commencing on 15 November 1978. The private
respondents then introduced additional improvements and registered the
house in their names. After the expiration of the lease in November 1985,
however, the petitioners' mother refused to accept the monthly rentals.It
turned out that the lot in question was the subject of a suit, which resulted in
its acquisition by one Maria Lee in 1972. In 1982, Lee sold the lot to Lily
Salcedo, who in turn sold it in 1984 to the spouses Agustin and Ester Dionisio.
RTC ruled that since the private respondents were assured by the petitioners
that the lot they leased would eventually be sold to them, they could be
considered builders in good faith, and as such, were entitled to
reimbursement of the value of the house and improvements with the right of
retention until reimbursement had been made.
ISSUE: Are the private respondents builders in good faith?
RULING:
NO. Being mere lessees, the private respondents knew that their occupation
of the premises would continue only for the life of the lease. Plainly, they
cannot be considered as possessors nor builders in good faith. This Court has
held that Article 448 of the Civil Code, in relation to Article 546 of the same
Code, which allows full reimbursement of useful improvements and retention
of the premises until reimbursement is made, applies only to a possessor in
good faith, i.e., one who builds on land with the belief that he is the owner
thereof. It does not apply where one's only interest is that of a lessee under a
rental contract; otherwise, it would always be in the power of the tenant to
"improve" his landlord out of his property. And even if the petitioners indeed
promised to sell, it would not make the private respondents possessors or
builders in good faith so as to be covered by the provisions of Article 448 of
the Civil Code. The latter cannot raise the mere expectancy of ownership of
the aforementioned lot because the alleged promise to sell was not fulfilled
nor its existence even proven. The first thing that the private respondents
should have done was to reduce the alleged promise into writing, because
under Article 1403 of the Civil Code, an agreement for the sale of real
property or an interest therein is unenforceable, unless some note or
memorandum thereof be produced. Not having taken any steps in order that
the alleged promise to sell may be enforced, the private respondents cannot
47 4S 2014-2015

San Beda College of Law CIVIL LAW REVIEW



bank on that promise and profess any claim nor color of title over the lot in
question.

OWNERSHIP: Accession Natural; Alluvium or Alluvion


AGUSTIN v. INTERMEDIATE APPELLATE COURT
G.R. Nos. L-66075-76, 5 July 1990
[MEDINA]

FACTS:
The Cagayan River separates the towns of Solana on the west and
Tuguegarao on the east in the province of Cagayan. The land east of the
Cagayan River is owned by defendant-petitioner Eulogio Agustin (Tuguegarao
Cadastre). As the years went by, the Cagayan River moved gradually
eastward, depositing silt on the western bank. The shifting of the river and the
siltation started by 1919 and continued until 1968.
Through the years, the Cagayan River eroded lands of the Tuguegarao
Cadastre on its eastern bank among which was defendant-petitioner Eulogio
Agustin's, depositing the alluvium as accretion on the land possessed by
Pablo Binayug on the western bank.
However, in 1968, after a big flood, the Cagayan River changed its course,
returned to its 1919 bed, and, in the process, cut across the lands of Maria
Melad, Timoteo Melad, and the spouses Pablo Binayug and Geronima Ubina
whose lands were transferred on the eastern, or Tuguegarao, side of the river.
To cultivate those lots they had to cross the river.
In April, 1969, while the private respondents and their tenants were planting
corn on their lots located on the eastern side of the Cagayan River, the
petitioners, accompanied by the mayor and some policemen of Tuguegarao,
claimed the same lands as their own and drove away the private respondents
from the premises.
On April 21, 1970, private respondents filed a complaint to recover their lots
and their accretions.
ISSUE: Are private respondents entitled to recover their lots and their
accretions?
RULING:
YES. There had been accretions to the lots of the private respondents and
they did not lose the ownership of such accretions even after they were
separated from the principal lots by the sudden change of course of the river.
Art. 457 of the New Civil Code which provides that to the owners of lands
adjoining the banks of rivers belong the accretion which they gradually receive
from the effects of the current of the waters.
48 4S 2014-2015

San Beda College of Law CIVIL LAW REVIEW



Accretion benefits a riparian owner when the following requisites are present:
(1) that the deposit be gradual and imperceptible; (2) that it resulted from the
effects of the current of the water; and (3) that the land where accretion takes
place is adjacent to the bank of a river. All these requisites of accretion are
present in this case for the accretion on the western bank of the Cagayan
River had been going on from 1919 up to 1968 or for a period of 49 years. It
was gradual and imperceptible.
The private respondents' ownership of the accretion to their lands was not lost
upon the sudden and abrupt change of the course of the Cagayan River in
1968 or 1969 when it reverted to its old 1919 bed, and separated or
transferred said accretions to the other side (or eastern bank) of the river.
Articles 459 and 463 of the New Civil Code apply to this situation.
Art. 459. Whenever the current of a river, creek or torrent segregates from an estate on its
bank a known portion of land and transfers it to another estate, the owner of the land to which
the segregated portion belonged retains the ownership of it, provided that he removes the
same within two years.
Art. 463. Whenever the current of a river divides itself into branches, leaving a piece of land
or part thereof isolated, the owner of the land retains his ownership. He also retains it if a
portion of land is separated from the estate by the current. (Emphasis supplied).

In the case at bar, the sudden change of course of the Cagayan River as a
result of a strong typhoon in 1968 caused a portion of the lands of the private
respondents to be "separated from the estate by the current." The private
respondents have retained the ownership of the portion that was transferred
by avulsion to the other side of the river.
CUREG v. INTERMEDIATE APPELLATE COURT
G.R. No. 73465, 7 September 1989
[MENESES]
FACTS:
Private respondent Domingo Apostol bought a parcel of land from private
respondents Soledad Gerardo, Rosa Gerardo, Nieves Gerardo, Flordeliza
Gerardo (Gerardos), and Maquinad. At the time of the execution by the
vendors of an Extra-Judicial Partition of the property, the parcel of land
already showed signs of accretion of about three (3) hectares. Thereafter,
when private respondents were about to cultivate the land together with its
accretion, they were prevented and threatened by petitioner Cureg. Petitioner
Cureg anchors his claim on the parcel of land and its accretion on Original
Certificate of Title No. P-19093 (the OCT only covered the parcel of land),
while private respondents anchor their claim on previously filed tax
declarations stating the area and boundaries of the land.
[In sum, the Court ruled in favor of Cureg, the latter being the rightful owner of the parcel of land as well
as the accretion. Other facts are not anymore relevant to the issue that is related to Property. Actually,
the property-related issue in this case was only a minor one, was tackled by the SC at the end of the
decision, and only in passing].

49 4S 2014-2015

San Beda College of Law CIVIL LAW REVIEW



ISSUE: Is Cureg still required to register, under the Torrens System, the land
acquired through accretion, even if the parcel of land which received the
accretion was already covered by a Torrens title?
RULING:
YES. An increase in the area of ones land through an accretion left by the
change of course or the northward movement of a body of water does not
automatically become registered land just because the lot which receives
such accretion is covered by a Torrens title. As such, it must also be placed
under the operation of the Torrens System.
VIAJAR v. COURT OF APPEALS
G.R. No. L-77294, December 12, 1988
[ORSUA]
FACTS:
The spouses Ladrido were the owners of Lot No. 7511 of the Cadastral
Survey situated in barangay Cawayan, Pototan, Iloilo. This lot was registered
in the names of the spouses under TCT of the Register of Deeds of Iloilo.
Spouses Te were also the registered owners of a parcel of land described in
their title as Lot No. 7340 of the Cadastral Survey. Te sold this lot Spouses
Viajar. A Torrens title was later issued in their names. Later, Angelica Viajar
had Lot No. 7340 relocated and found out that the property was in the
possession of Ricardo Ladrido. Consequently, she demanded its return but
Ladrido refused.
Spouses Viajar instituted a civil action for recovery of possession and
damages against Ricardo Ladrido and Rosendo Te. Plaintiffs sought the
annulment of the deed of sale and the restitution of the purchase price with
interest in the event the possession of defendant Ladrido is sustained.
It is admitted by the parties that Lot No. 7511 and Lot No. 7340 were
separated by the Suague River. Petitioners also contend that Article 457 of
the New Civil Code must be construed to limit the accretion mentioned therein
as accretion of unregistered land to the riparian owner, and should not extend
to registered land. The lower court and CA ruled in favour of defendants.
ISSUES:
1. Is the change in the course of the Suague River sudden as claimed by the
plaintiffs in order to negate the right of accretion by the defendants?
2. Is accretion under Art. 457 limited to unregistered lands, as such protecting
registered lands against diminution in area?
RULING:
1. NO. For a period of more than 40 years (before 1940 to 1980) the Suague
River overflowed its banks yearly and the property of the defendant gradually
received deposits of soil from the effects of the current of the river. The
consequent increase in the area of Lot No. 7511 due to alluvion or accretion
was possessed by the defendants whose tenants plowed and planted the
same with corn and tobacco. Art. 457 provides that to the owners of lands
50 4S 2014-2015

San Beda College of Law CIVIL LAW REVIEW



adjoining the banks of rivers belong the accretion which they gradually receive
from the effects of the current of the waters.
2. NO. It also applies to registered lands. The rule that registration under the
Torrens System does not protect the riparian owner against the diminution of
the area of his registered land through gradual changes in the course of an
adjoining stream is well settled.
VDA. DE NAZARENO v. COURT OF APPEALS
G.R. No. 98045, 26 June 1996
[PAGAYANAN]
FACTS:
The subject of this controversy is a parcel of land situated in Telegrapo,
Puntod, Cagayan de Oro City. Said land was formed as a result of sawdust
dumped into the dried-up Balacanas Creek and along the banks of the
Cagayan River. Sometime in 1979, private respondents Jose Salasalan and
Leo Rabaya leased the subject lots on which their houses stood from one
Antonio Nazareno, petitionerspredecessor-in-interest. In the latter part of
1982, private respondents allegedly stopped paying rentals. As a result,
Antonio Nazareno and petitioners filed a case for ejectment with the MTC of
Cagayan de Oro. A decision was rendered against private respondents, which
decision was affirmed by the RTC of Misamis Oriental.
The case was remanded to the municipal trial court for execution of judgment
after the same became final and executory. Private respondents filed a case
for annulment of judgment before the RTC of Misamis Oriental, which
dismissed the same. Subsequently, the decision of the lower court was finally
enforced with the private respondents being ejected from portions of the
subject lots they occu
Before he died, Antonio Nazareno caused the approval by the Bureau of
Lands of the survey plan with a view to perfecting his title over the accretion
area being claimed by him. Before the approved survey plan could be
released to the applicant, however, it was protested by private respondents
before the Bureau of Lands. In compliance with the order of respondent
District Land Officer Alberto M. Gillera, respondent Land Investigator Avelino
G. Labis conducted an investigation and rendered a report to the Regional
Director recommending that the survey plan in the name of Antonio Nazareno,
be cancelled and that private respondents be directed to file appropriate
public land applications.
Based on said report, respondent Regional Director of the Bureau of Lands
Roberto Hilario rendered a decision ordering the amendment of the survey
plan in the name of Antonio Nazareno by segregating therefrom the areas
occupied by the private respondents who, if qualified, may file public land
applications covering their respective portions.

51 4S 2014-2015

San Beda College of Law CIVIL LAW REVIEW



ISSUE: Is the subject private land, which was formed as a result of a sawdust
dumped into the dried-up creek and along the banks of a river, can be
considered as alluvion?
RULING:
NO. Court held that accretion, as a mode of acquiring property under Art. 457
of the Civil Code, requires the concurrence of these requisites: (1) that the
deposition of soil or sediment be gradual and imperceptible; (2) that it be the
result of the action of the waters of the river (or sea); and (3) that the land
where accretion takes place is adjacent to the banks of rivers (or the sea
coast). These are called the rules on alluvion which if present in a case, give
to the owners of lands adjoining the banks of rivers or streams any accretion
gradually received from the effects of the current of waters.
For petitioners to insist on the application of these rules on alluvion to their
case, the above-mentioned requisites must be present. However, they admit
that the accretion was formed by the dumping of boulders, soil and other filling
materials on portions of the Balacanas Creek and the Cagayan River
bounding their land. It cannot be claimed, therefore, that the accumulation of
such boulders, soil and other filling materials was gradual and imperceptible,
resulting from the action of the waters or the current of the Balacanas Creek
and the Cagayan River.
The accretion was man-made or artificial. The requirement that the deposit
should be due to the effect of the current of the river is indispensable. This
excludes from Art. 457 of the Civil Code all deposits caused by human
intervention. Putting it differently, alluvion must be the exclusive work of
nature. In the case at bar, the subject land was the direct result of the
dumping of sawdust by the Sun Valley Lumber Co. consequent to its sawmill
operations.
HEIRS OF EMILIANO NAVARRO v. INTERMEDIATE APPELLATE COURT
G.R. No. 68166, 12 February 1997
[PARUNGAO]
FACTS:
Sinforoso Pascual filed an application to register and confirm his title to a
parcel of land, situated in Sibocon, Balanga, Bataan. Pascual claimed that this
land is an accretion to his property, situated in Barrio Puerto Rivas, Balanga,
Bataan. It is bounded on the eastern side by the Talisay River, on the western
side by the Bulacan River, and on the northern side by the Manila Bay. The
Talisay River as well as the Bulacan River flow downstream and meet at the
Manila Bay thereby depositing sand and silt on Pascual's property resulting in
an accretion thereon. Sinforoso Pascual claimed the accretion as the riparian
owner.
Navarro thereupon filed an opposition to Pascual's application claiming that
the land sought to be registered has always been part of the public domain, it
being a part of the foreshore of Manila Bay; that he was a lessee and in
52 4S 2014-2015

San Beda College of Law CIVIL LAW REVIEW



possession of a part of the subject property and that he had already converted
the area covered by the lease into a fishpond.
The case was decided adversely against Pascual. Thus, Pascual appealed to
the Court of First Instance (now Regional Trial Court) of Balanga, Bataan.
During the pendency of the trial of the consolidated cases, Emiliano Navarro
and Pascual died and were substituted by their heirs.
The court a quo rendered judgment finding the subject property to be
foreshore land and, being a part of the public domain, it cannot be the subject
of land registration proceedings. The heirs of Pascual appealed before the
respondent appellate court which reversed the findings of the court a quo and
granted the petition for registration of the subject property.
ISSUE: Whether the disputed land is an accretion caused by the joint action of
the Talisay and Bulacan Rivers which run their course on the eastern and
western boundaries, respectively, of petitioners' own tract of land.
RULING:
Accretion as a mode of acquiring property under said Article 457, requires the
concurrence of the following requisites: (1) that the accumulation of soil or
sediment be gradual and imperceptible; (2) that it be the result of the action of
the waters of the river; and (3) that the land where the accretion takes place is
adjacent to the bank of the river. Accretion is the process whereby the soil is
deposited, while alluvium is the soil deposited on the estate fronting the river
bank; the owner of such estate is called the riparian owner. Riparian owners
are, strictly speaking, distinct from littoral owners, the latter being owners of
lands bordering the shore of the sea or lake or other tidal waters. The
alluvium, by mandate of Article 457 of the Civil Code, is automatically owned
by the riparian owner from the moment the soil deposit can be seen but is not
automatically registered property, hence, subject to acquisition through
prescription by third persons.

CO-OWNERSHIP: Rights of Each Co-owner



DEL BANCO v. INTERMEDIATE APPELLATE COURT
G.R. No. 72694, 1 December 1987
[PULMA]
FACTS:
Three brothers, Benedicto Pansacola, Jose Pansacola and Manuel Pansacola
(known as Fr. Manuel Pena) entered into an agreement which provided,
among others:
(1) That they will purchase from the Spanish Government the lands
comprising the Island of Cagbalite which is located within the boundaries of
the Municipality of Mauban, Province of Tayabas (now Quezon) and has an
approximate area of 1,600 hectares;
53 4S 2014-2015

San Beda College of Law CIVIL LAW REVIEW



(2) That the lands shall be considered after the purchase as their common
property;
(3) That the co-ownership includes Domingo Arce and Baldomera Angulo,
minors at that time represented by their father, Manuel Pansacola (Fr. Manuel
Pena) who will contribute for them in the proposed purchase of the Cagbalite
Island;
(4) That whatever benefits may be derived from the Island shall be shared
equally by the co-owners in the following proportion: Benedicto Pansacola-1/4
share; Jose Pansacola-1/4 share; and, Domingo Arce and Baldomera Angulo2/4 shares which shall be placed under the care of their father, Manuel
Pansacola (Fr. Manuel Pena).
Co-owners entered into the actual possession and enjoyment of the Island
purchased by them from the Spanish Government. They agreed to modify the
terms and conditions of the agreement entered into by them on. The new
agreement provided for a new sharing and distribution of the lands,
comprising the Island of Cagbalite and whatever benefits may be derived
therefrom. About one hundred years later, private respondents brought a
special action for partition in the Court of First Instance of Quezon. Petitioners
herein, interposed such defenses as prescription, res judicata, exclusive
ownership, estoppel and laches.
ISSUE: Can mere agreement to subdivide property terminate co-ownership?
RULING:
NO. It is not enough that the co-owners agree to subdivide the property. They
must have a subdivision plan drawn in accordance with which they take actual
and exclusive possession of their respective portions in the plan and titles
issued to each of them accordingly (Caro v. Court of Appeals, 113 SCRA 10
[1982]). The mechanics of actual partition should follow the procedure laid
down in Rule 69 of the Rules of Court. Maganon v. Montejo, 146 SCRA 282
[1986]).
Neither can such actual possession and enjoyment of some portions of the
Island by some of the petitioners herein be considered a repudiation of the coownership. It is undisputed that the Cagbalite Island was purchased by the
original co-owners as a common property and it has not been proven that the
Island had been partitioned among them or among their heirs. While there is
co-ownership, a co-owner's possession of his share is co-possession which is
linked to the possession of the other co-owners (Gatchalian v. Arlegui, 75
SCRA 234 [1977]).
Furthermore, no prescription shall run in favor of a co-owner against his coowners or co-heirs so long as he expressly or impliedly recognizes the coownership (Valdez v. Olonga, 51 SCRA 71 [1973], Tero v. Tero, 131 SCRA
100 [1984]). Co-owners cannot acquire by prescription the share of the other
co-owners, absent a clear repudiation of the co-ownership clearly
54 4S 2014-2015

San Beda College of Law CIVIL LAW REVIEW



communicated to the other co-owners (Mariano v. De Vega, 148 SCRA 342
[1987]).
An action for partition does not prescribe. Article 403 of the Old Civil Code,
now Article 497, provides that the assignees of the co-owners may take part
in the partition of the common property, and Article 400 of the Old Code, now
Article 494 provides that each co-owner may demand at any time the partition
of the common property, a provision which implies that the action to demand
partition is imprescriptible or cannot be barred by laches (Budlong v. Pondoc,
79 SCRA 24 [1977]). An action for partition does not lie except when the coownership is properly repudiated by the co- owner (Jardin v. Hollasco, 117
SCRA 532 [1982]).

CO-OWNERSHIP: Obligations of Each Co-owner


PARDELL v. BARTOLOME
G.R. No. L-4656, 18 November 1912
[REMIGIO]

FACTS:
Petitioner Vicenta Ortiz y Felin de Pardell and respondent Matilde Ortiz y Felin
Bartolome were the existing heirs of the late Miguel Ortiz and Calixta Felin.
On 1888, Matilde and co-defendant Gaspar de Bartolome y Escribano took it
upon themselves without an judicial authorization or even extra judicial
agreement the administration of the properties of the late Calixta and Miguel.
These properties included a house in Escolta Street, Vigan, Ilocos Sur; a
house in Washington Street, Vigan, Ilocos Sur; a lot in Magallanes Street,
Vigan, Ilocos Sur; parcels of rice land in San Julian and Sta. Lucia; and
parcels of land in Candon, Ilocos Sur.
Vicenta filed an action in court asking that the judgement be rendered in
restoring and returning to them one half of the total value of the fruits and
rents, plus losses and damages from the aforementioned properties.
However, respondent Matilde asserted that she never refused to give the
plaintiff her share of the said properties. Vicenta also argued that Matilde and
her husband, Gaspar are obliged to pay rent to the former for their occupation
of the upper story of the house in Escolta Street.
ISSUE: Is defendant Matilde Ortiz, as co-owner of the house on Calle Escolta,
entitled, with her husband, to reside therein, without paying to her co-owner,
Vicenta Ortiz, one-half of the rents which the upper story would have
produced, had it been rented to a stranger?
RULING:
NO. The Court ruled that the spouses are not liable to pay rent. Their
occupation of the said property was a mere exercise of their right to use the
same as a co-owner. Each co-owner or tenant in common of undivided realty
has the same rights therein as the others; he may use and enjoy the same
55 4S 2014-2015

San Beda College of Law CIVIL LAW REVIEW



without other limitation except that he must not prejudice the rights of his coowners, but until a division is effected, the respective parts belonging to each
can not be determined; each co-owner exercises joint dominion and is entitled
to joint use.
Matilde Ortiz and her husband occupied the upper story, designed for use as
a dwelling, in the house of joint ownership; but the record shows no proof that,
by so doing, Matilde occasioned any detriment to the interests of the
community property, nor that she prevented her sister Vicenta from utilizing
the said upper story according to her rights. It is to be noted that the stores of
the lower floor were rented and an accounting of the rents was duly made to
the plaintiffs.

CO-OWNERSHIP: Extinguishment of Co-ownership


CARO v. COURT OF APPEALS
G.R. No. L-48001, 25 March 1982
[RAMPAS]

FACTS:
A parcel of land was co-owned by three siblings. One sold his undivided share
to RP. Co-owners did not exercise their right of legal redemption, but widow of
a deceased co-owner, as administratrix, moved for right of legal redemption.
ISSUE: Can an administratrix of a deceased co-owner exercise right of legal
redemption on the theory that the property has already been agreed to be
divided among the co-owners?
RULING:
Once a property is partitioned, co-ownership is terminated and property
ceases to be community.
Inasmuch as the purpose of the law in establishing the right of legal
redemption between co-owners is to reduce the number of participants until
the community is done away with (Viola v. Tecson, 49 Phil. 808), once the
property is subdivided and distributed among the co-owners, the community
has terminated and there is no reason to sustain any right of legal redemption.
BAILON-CASILAO v. COURT OF APPEALS
No. L-78178, 15 April 1988
[SABILALA]
FACTS:
The petitioners herein filed a case for recovery of property and damages with
notice of lis pendens on March 13, 1981 against the defendant and herein
private respondent, Celestino Afable. The parcel of land involved in this case
in the names of Rosalia, Gaudencio, Sabina, Bernabe, Nenita and Delia, all
56 4S 2014-2015

San Beda College of Law CIVIL LAW REVIEW



surnamed Bailon, as co-owners, each with a 1/6 share. Gaudencio and Nenita
are now dead, the latter being represented in this case by her children. Luz,
Emma and Nilda. Bernabe went to China in 1931 and had not been heard
from since then.
It appears that on August 23, 1948, Rosalia Bailon and Gaudencio Bailon sold
a portion of the said land consisting of 16,283 square meters to Donato
Delgado. On May 13, 1949, Rosalia Bailon alone sold the remainder of the
land consisting of 32,566 square meters to Ponciana V. Aresgado de Lanuza.
On the same date, Lanuza acquired from Delgado the 16,283 square meters
of land which the latter had earlier acquired from Rosalia and Gaudencio. On
December 3, 1975, John Lanuza, acting under a special power of attorney
given by his wife, Ponciana V. Aresgado de Lanuza, sold the two parcels of
land to Celestino Afable, Sr.
In his answer to the complaint filed by the herein petitioners, Afable claimed
that he had acquired the land in question through prescription and contended
that the petitioners were guilty of laches.
ISSUE: Are petitioners guilty of laches as may effectively bar their present
action?
RULING:
NO. Since a co-owner is entitled to sell his undivided share, a sale of the
entire property by one co-owner without the consent of the other co-owners is
not null and void. However, only the rights of the co-owner-seller are
transferred, thereby making the buyer a co-owner of the property. The proper
action in cases like this is not for the nullification of the sale or for the recovery
of possession of the thing owned in common from the third person who
substituted the co-owner or co-owners who alienated their shares, but the
DIVISION of the common property as if it continued to remain in the
possession of the co-owners who possessed and administered it. Neither
recovery of possession nor restitution can be granted since the defendant
buyers are legitimate proprietors and possessors in joint ownership of the
common property claimed.
In the light of the attendant circumstances, defendant-appellees defense of
prescription is a vain proposition. Pursuant to Article 494 of the Civil Code,
"(n)o co-owner shall be obliged to remain in the co-ownership. Such co-owner
may demand at anytime the partition of the thing owned in common, insofar
as his share is concerned. In Budlong v. Bondoc [G.R. No. L-27702,
September 9, 1977, 79 SCRA 24], this Court has interpreted said provision of
law to mean that the action for partition is imprescriptible or cannot be barred
by prescription. For Article 494 of the Civil Code explicitly declares: No
prescription shall lie in favor of a co-owner or co-heir so long as he expressly
or impliedly recognizes the co-ownership.
As regards with Laches, it has been defined as the failure or neglect, for an
unreasonable length of time to do that which by exercising due diligence could
57 4S 2014-2015

San Beda College of Law CIVIL LAW REVIEW



or should have been done earlier; it is negligence or omission to assert a right
within a reasonable time warranting a presumption that the party entitled to
assert it either has abandoned it or declined to assert it. It must be noted that
while there was delay in asserting petitioners rights, such delay was not
attended with any knowledge of the sale nor with any opportunity to bring
suit. In view of the lack of knowledge by the petitioners of the conduct of
Rosalia in selling the land without their consent in 1975 and the absence of
any opportunity to institute the proper action until 1981, laches may not be
asserted against the petitioners.
ROQUE v. INTERMEDIATE APPELLATE COURT
G.R. No. 75886, 30 August 1988
[SANTILLAN]
FACTS:
The controversy here involves a parcel of land which was originally in the
name of Januario Avendao, a bachelor who died intestate and without issue.
The intestate heirs of Avendafio executed a document through which an
extrajudicial partition was effected among the intestate heirs. Some of the coowners then, all surnamed Avendao, transferred their undivided threefourths
(3/4) share to respondent Ernesto Roque and Victor Roque, thereby vesting in
the latter full and complete ownership of the property. Subsequently, in an
unnotarized deed Emesto and Victor Roque purportedly sold a three-fourths
(3/4) undivided portion of the subject land to their half-sister, petitioner
Concepcion Roque. The property, however, remained registered in the name
of the decedent, Januario Avendao. Upon the instance of petitioner and
allegedly of respondent the land was surveyed. Consequent thereto, a
Subdivision Plan was drawn. Petitioner claimed that preparation of the
Subdivision Plan was a preliminary step leading eventually to partition.
Respondents, however, refused to acknowledge petitioner's claim of
ownership of any portion of Lot No. 1549 and rejected the plan to divide the
land since the respondents alleged that they are the absolute owner of the
subject land and that petitioner occupies a portion of the said property only by
their mere tolerance.
ISSUE: Does the right to seek an action for partition of the thing owned in
common prescribes?
RULING:
NO. According to Article 494 of the Civil Code, "each co-owner may demand
at any time the partition of the thing owned in common, insofar as his share is
concemed." No matter how long the co-ownership has lasted, a co-owner can
always opt out of the co-ownership, and provided the defendant co-owners or
co-heirs have theretofore expressly or impliedly recognized the co-ownership,
they cannot set up as a defense the prescription of the action for partition. But
if the defendants show that they had previously asserted title in themselves
adversely to the plaintiff and for the requisite period of time, the plaintiffs right
to require recognition of his status as a co-owner will have been lost by
prescription and the court cannot issue an order requiring partition.
58 4S 2014-2015

San Beda College of Law CIVIL LAW REVIEW



In the case at bar, First of all, petitioner Concepcion Roque-the co-owner
seeking partition has been and is presently in open and continuous
possession of a three-fourths (3/4) portion of the property owned in common.
Second, prior to filing of the Civil Case, neither of the parties involved had
asserted or manifested a claim of absolute and exclusive ownership over the
whole of Lot No. 1549 adverse to that of any of the other co-owners: in other
words, co-ownership of the property had continued to be recognized by all the
owners. Consequently, the action for partition could not have and, as a matter
of fact, had not yet prescribed at the time of institution by Concepcion of the
action.
DELIMA v. COURT OF APPEALS
G.R. No. L-46296, 24 September 1991
[SAYO]
FACTS:
During his lifetime, Lino Delima acquired Lot No. 7758. Lino Delima later died
leaving as his only heirs three brothers and a sister namely: Eulalio, Juanita,
Galileo and Vicente, all surnamed Delima. After his death, TCT No. 2744 of
the property in question was issued in the name of the Legal Heirs of Lino
Delima, deceased represented by Galileo Delima.
Galileo Delima, now substituted by respondents, executed an affidavit of
Extra-judicial declaration of Heirs. Based on this affidavit, TCT No. 2744 was
cancelled and TCT No. 3009 was issued on February 4, 1954 in the name of
Galileo Delima alone to the exclusion of the other heirs.
On February 29, 1968, petitioners filed an action for reconveyance and/or
partition and for the annulment of TCT No. 3009 with damages. The RTC
ruled in favor of petitioners, however on appeal, the CA reversed the decision
of the RTC. Hence, this petition for review on certiorari.
ISSUE: Is the action already barred by prescription, which shall entitle Galileo
Delima to perfect his claim of ownership by acquisitive prescription to the
exclusion of petitioners?
RULING:
YES. From the moment one of the co-owners claims that he is the absolute
and exclusive owner of the properties and denies the others any share
therein, the question involved is no longer one of partition but of ownership. In
such a case, the imprescriptibility of the action for partition can no longer be
invoked.
It is settled that possession by a co-owner or co-heir is that of a trustee. In
order that such possession is considered adverse to the cestui que trust
amounting to a repudiation of the co-ownership, the following elements must
concur: 1) that the trustee has performed unequivocal acts amounting to an
ouster of the cestui que trust; 2) that such positive acts of repudiation had
59 4S 2014-2015

San Beda College of Law CIVIL LAW REVIEW



been made known to the cestui que trust; and 3) that the evidence thereon
should be clear and conclusive.
Since an action for reconveyance of land based on implied or constructive
trust prescribes after ten (10) years, it is from the date of the issuance of such
title that the effective assertion of adverse title for purposes of the statute of
limitations is counted, or in this case from February 4, 1954.
As the certificate of title was notice to the whole world of his exclusive title to
the land, such rejection was binding on the heirs and started as against them
the period of prescription. Hence, when petitioners filed their action for
reconveyance on February 29, 1968, such action was already barred by
prescription. Whatever claims the other co-heirs could have validly asserted
before can no longer be invoked by them at this time.
AGUILAR v. COURT OF APPEALS
G.R. No. 76351, 29 October 1993
[SENTILLAS]
FACTS:
Virgilio and Senen are brothers; Virgilio is the youngest of seven (7) children
of the late Maximiano Aguilar, while Senen is the fifth. On 28 October 1969,
the two brothers purchased a house and lot in Paraaque where their father
could spend and enjoy his remaining years in a peaceful neighborhood.
Initially, the brothers agreed that Virgilio's share in the co-ownership was twothirds while that of Senen was one-third. By virtue of a written memorandum,
Virgilio and Senen agreed that henceforth their interests in the house and lot
should be equal, with Senen assuming the remaining mortgage obligation of
the original owners with the Social Security System (SSS) in exchange for his
possession and enjoyment of the house together with their father.
Since Virgilio was then disqualified from obtaining a loan from SSS, the
brothers agreed that the deed of sale would be executed and the title
registered in the meantime in the name of Senen. It was further agreed that
Senen would take care of their father and his needs since Virgilio and his
family were staying in Cebu.
After Maximiano Aguilar died in 1974, Virgilio demanded from Senen that the
latter vacate the house and that the property be sold and proceeds thereof
divided among them.
Senen refused, prompting Virgilio to file an action to compel the sale of the
house and lot so that the they could divide the proceeds between them.
The trial court found Virgilio and Senen to be co-owners of the house and lot,
in equal shares on the basis of their written agreement. However, it ruled that
Virgilio has been deprived of his participation in the property by Senen's
continued enjoyment of the house and lot, free of rent, despite demands for
rentals and continued maneuvers to delay partition. The trial court also upheld
the right of Virgilio as co-owner to demand partition. Since Virgilio could not
60 4S 2014-2015

San Beda College of Law CIVIL LAW REVIEW



agree to the amount offered by Senen for the former's share, the trial court
held that this property should be sold to a third person and the proceeds
divided equally between the parties.
The trial court likewise ordered defendant to vacate the property and pay
plaintiff P1,200.00 as rentals from January 1975 up to the date of decision
plus interest from the time the action was filed.
ISSUES:
1.May a co-owner demand the sale of a house and lot held in common with
other co-owners?
2. Should a co-owner who enjoys possession of the thing held in common pay
rentals to other co-owners?
RULING:
1. YES. Article 494 of the Civil Code provides that no co-owner shall be
obliged to remain in the co-ownership, and that each co-owner may demand
at any time partition of the thing owned in common insofar as his share is
concerned. Corollary to this rule, Art. 498 of the Code states that whenever
the thing is essentially, indivisible and the co-owners cannot agree that it be,
allotted to one of them who shall indemnify the others, it shall be sold and its
proceeds accordingly distributed. This is resorted to a. when the right to partition the property is invoked by any of the
co-owners but because of the nature of the property it cannot be
subdivided or its subdivision would prejudice the interests of the
co-owners, and
b. the co-owners are not in agreement as to who among them shall
be allotted or assigned the entire property upon proper
reimbursement of the co-owners.
2. NO. Each co-owner of property held pro indiviso exercises his rights over
the whole property and may use and enjoy the same with no other limitation
than that he shall not injure the interests of his co-owners, the reason being
that until a division is made, the respective share of each cannot be
determined and every co-owner exercises, together with his co-participants
joint ownership over the pro indiviso property, in addition to his use and
enjoyment of the same.
ADDITIONAL FACTS PECULIAR TO THE CASE:
Although the Court did rule that each co-owner of property... may use and enjoy the same
with no other limitation than that he shall not injure the interests of his co-owners, the
respondent was still made to pay monthly rentals because he failed to vacate the house and
lot despite order of the trial court. He was ordered to pay rentals from the time the trial court
ordered him to vacate for the use and enjoyment of the other half of the property appertaining
to petitioner.

61 4S 2014-2015

San Beda College of Law CIVIL LAW REVIEW



TOMAS CLAUDIO MEMORIAL COLLEGE v. COURT OF APPEALS
G.R. No. 124262, 12 October 1999
[SINOCRUZ]
FACTS:
Private respondents De Castros filed an action for Partition before the RTC
alleging that their predecessor-in-interest, Juan, died intestate in 1993 and
they are his only surviving legitimate heirs. They also alleged that their father
owned a parcel of land designated as Lot No. 3010 in Morong, Rizal. They
further claimed that without their knowledge and consent, said lot was sold by
their brother Mariano to petitioner Tomas Claudio Memorial College (TCMC)
when the former represented himself to be the sole heir. The De Castros
contend that the sale should only affect Marianos undivided share but not the
shares of the other co-owners.
TCMC filed a motion to dismiss contending lack of jurisdiction and prescription
and/or laches.
ISSUE: Are petitioners De Castros right to partition barred by prescription?
RULING:
NO. Article 493 of the Civil Code provides that the sale or other disposition
affects only the sellers share pro indiviso, and the transferee gets only what
correspondents to his grantors share in the partition owned in
common. Since a co-owner is entitled to sell his undivided share, a sale of
the entire property by one co-owner without the consent of the other coowners is not null and void. However, only the rights of the co-owner/seller
are transferred, making the buyer a co-owner. The proper action in a case
like this is the division or partition of the entire property that continued to
remain in the possession of the co-owners.
With respect to partition, Article 494 of the Civil Code provides that no coowner shall be obliged to remain in the co-ownership. Such co-owner may
demand at any time the partition of the thing owned in common, insofar as his
share is concerned. xxx. No prescription shall lie in favor of a co-owner or coheirs as long as he expressly or impliedly recognizes that co-ownership.
ROBLES v. COURT OF APPEALS
G.R. Number 123509, 14 March 2000
[TILOS]
FACTS:
The property subject of this case is originally owned by Leon Robles. When
he died, it passed to his son Silvino who declared the property in his name
and paid the taxes thereon. Upon the latters death, his widow and children
inherited the property. Petitioners Lucio Robles, et al. were the children of
Silvino, and Hilario Robles is their half-brother. The task of cultivating was
assigned to Lucio while the payment of the land taxes was entrusted to
Hilario. For unknown reason, the tax declaration of the parcel of land in the
62 4S 2014-2015

San Beda College of Law CIVIL LAW REVIEW



name of Silvino was cancelled and transferred to Exequiel Ballena. Ballena
secured a loan from Antipolo Rural Bank using the tax declaration as
security. Somehow the tax declaration was transferred to the name of
Antipolo Rural Bank and later was transferred to the name of respondentspouses Hilario and Andrea Robles. Andrea secured a loan from
Cardona Rural Bank using the tax declaration as security. For failure to pay
the mortgage debt, the property was foreclosed with Cardona Rural
Bank emerging as the highest bidder. The bank sold the property to
spouses Vergel and Ruth Santos. In Sept. 1987, petitioners discovered the
mortgage and attempted to redeem the property but was unsuccessful. In
1988, the spouses Santos took possession of the propertry and was able to
secure a Free Patent. Petitioners then filed an action for quieting of
title. Respondents questioned their standing to sue for quieting of title,
contending that petitioners no longer have any interest to the property in
question due to the mortgage effected by Hilario and the consequent
foreclosure thereof by the Bank. Respondents argued that Hilario had
become the absolute owner of the property at the time he mortgaged the
same. The CA ruled that the several transfers of the tax declaration of the
property in question from Silvino until to the spouses Santos had the effect of
divesting petitioners of their title by prescription to Hilario.
ISSUES:
1. Do the petitioners have appropriate title that will entitle them to the remedy
of the quieting of title?
2. Did Hilario acquire the share of his co-owners in the disputed property by
prescription?
RULING:
1. YES. An action to quiet title, under Art. 476, NCC, is a common-law
remedy for the removal of any cloud or doubt or uncertainty on the title to real
property. It is essential for the plaintiff or complainant to have a legal or an
equitable title to or interest in the real property which is the subject matter of
the action. Also, the deed, claim, encumbrance or proceeding that is being
alleged as a cloud on plaintiff's title must be shown to be in fact invalid or
inoperative despite its prima facie appearance of validity or legal efficacy.
That there is an instrument or a document which, on its face, is valid and
efficacious is clear in the present case. Petitioners allege that their title as
owners and possessors of the disputed property is clouded by the tax
declaration and, subsequently, the free patent thereto granted to Spouses
Santos. Petitioners anchor their claim on their open and continuous
possession as owners. Spouses Santos, on the other hand, trace their claims
to Exequiel, and then to Hilario who mortgaged the same to the Bank as
absolute owner. It was from Exequiel that Hilarios claim is rooted. However,
in this case, there is a failure to show Exequiels title to the property in
question. When Hilario, therefore, mortgaged the property, he did so in his
capacity as mere co-owner thereof. Consequently, the said transaction did
not divest the petitioner of the title to the property at the time of the institution
of the complaint for quieting of title.
63 4S 2014-2015

San Beda College of Law CIVIL LAW REVIEW



2. NO. Hilario effected no clear and evident repudiation of the co-ownership. It
is a fundamental principle that a co-owner cannot acquire by prescription the
share of the other co-owners, absent any clear repudiation of the coownership. In order that the title may prescribe in favor of a co-owner, the
following requisites must concur: (1) the co-owner has performed unequivocal
acts of repudiation amounting to an ouster of the other co-owners; (2) such
positive acts of repudiation have been made known to the other co-owners;
and (3) the evidence thereof is clear and convincing. In the present case,
Hilario did not have possession of the subject property; neither did he exclude
the petitioners from the use and the enjoyment thereof, as they had
indisputably shared in its fruits. Likewise, his act of entering into a mortgage
contract with the bank cannot be construed to be a repudiation of the coownership. As absolute owner of his undivided interest in the land, he had the
right to alienate his share, as he in fact did. Neither should his payment of
land taxes in his name, as agreed upon by the co-owners, be construed as a
repudiation of the co-ownership. The assertion that the declaration of
ownership was tantamount to repudiation was belied by the continued
occupation and possession of the disputed property by the petitioners as
owners.

POSSESSION: Classifications of Possession


RIZAL CEMENT CO., INC., v. CONSUELO C. VILLAREAL, ISABEL C.
VILLAREAL, FLAVIANO C. VILLAREAL, ALFREDO V. GOMEZ, AURORA
V. GOMEZ AND THE COURT OF APPEALS
Gr. No. L-30272, 28 February 1985
[ABIERA]

FACTS:
Private respondents filed with the then Court of First Instance of Rizal in
Pasig, an Application for Registration, alleging that they are the owners of two
agricultural lots bounded and described as shown on plan Psd-147662 as
Lots Nos. 1 and 2. Based on respondents' testimonial and documentary
evidence, it appears that the subject lots were originally belonged to one
Maria Certeza; that upon her death, the property was involved in a litigation
between her grandchildren and Gonzalo Certeza and that the lots were given
by the latter to former Justice de Joya as the latter's attorney's fees; that the
lots were then sold by de Joya to Filomeno Sta. Ana who, in turn sold the
same to spouses Victoriano Cervo and Ignacia Guillermo; and that sometime
in November 1955, the said spouses sold the said lots to the herein
respondents as shown by a duly notarized deed of sale.
On the other hand, petitioner Rizal Cement Company filed an opposition,
claiming to be the owner of the subject lots, having bought the same from
Maria Certeza, and to have been in continuous and adverse possession of the
property since 1911. To substantiate its claim, petitioner submitted
documentary evidence, the most important of which are the following 1)
Plan Psu-2260 covering the subject lots; 2) A sketch plan of the geographical
64 4S 2014-2015

San Beda College of Law CIVIL LAW REVIEW



position of the real pro-parties of Madrigal and Company; 3) Tax declaration;
and 4) Real estate receipts.
During trial, witnesses for the respondents testified that petitioner did not take
possession of the land and that it was Maria Certeza who had the possession
of the land until her death and that the tenants gave the harvest of the land to
Maria Certeza.
After trial, the CFI denied the application for registration of respondents and
ordered the issuance of a decree of registration in the name of Rizal Cement
Co. after finality of said decision. Respondents appealed to the Court of
Appeals, which reversed and set aside the CFIs decision in favor of the
respondents. The Court of Appeals denied Rizals Motion for Reconsideration,
hence this petition.
ISSUE: Did the respondents possess the property in the concept of an owner?
RULING:
YES. Being an attribute of ownership, appellants' possession of the land in
question goes far to tip the scale in their favor. The right to possess flows from
ownership. No person will suffer adverse possession by another of what
belongs to him. Were the petitioner rightful owner of the land in question, it
would not have allowed the tenants to cultivate the land and give the owner's
share to appellants and/or their predecessors. It would have opposed the
survey for respondents' vendors but did not as shown in the surveyor's
certificate. If petitioner really bought Lot 2 from Maria Certeza as claimed, it
has not been explained how she could sell a portion thereof to Apolonia
Francisco, married to Valentin Marquez, as confirmed by the husband in his
deposition who as employee of oppositor would have known of its acquisition.
As correctly held by the Court of Appeals, respondents possess the property
in the concept of an owner. Possession is acquired by the material occupation
of a thing or the exercise of a right or by the fact it is subject to the action of
our will, or by the proper acts and legal formalities established for acquiring
such right. Petitioner's evidence, consisting of tax receipts, tax declaration
and survey plan are not conclusive and indisputable basis of one's ownership
of the property in question. Assessment alone is of little value as proof of title.
Mere tax declaration does not vest ownership of the property upon the
declarant. Settled is the rule that neither tax receipts nor declaration of
ownership for taxation purposes alone constitutes sufficient evidence of
ownership or of the right to possess realty. They must be supported by other
effective proofs. Neither can the survey plan or technical descriptions
prepared at the instance of the party concerned be considered in his favor,
the same being self-serving.

65 4S 2014-2015

San Beda College of Law CIVIL LAW REVIEW



IGNACIO WONG v. HON. LUCAS D. CARPIO
G.R. No. L-50264, 21 October 1991
[ALDUESO]

FACTS:
The CFI of Davao Del Sur presided by the public respondent reversed the
ruling of the Municipal Court of Sta. Maria, Davao del Sur in an action for
forcible entry (FE) ordering the dismissal of the complaint as well as the
counterclaim.
Manuel Mercado (herein private respondent, plaintiff in lower court) acquired
his rights to possess the land in litigation which is particularly described and
embraced in Transfer Certificate of title No. (T-4244) T-972 from William Giger
by virtue of a deed of sale with right to repurchase which was executed in
1972 for a consideration of P3,500.00. Then, in 1973, William Giger again
asked an additional amount of P2,500.00 from plaintiff and so he required
William Giger to sign a new deed of Pacto de Retro Sale. In 1972, plaintiff
began harvesting only the coconut fruits and he paid the taxes on the landfor
Mr. Giger. He went periodically to the land to make copra but he never placed
any person on the land in litigation to watch it. Neither did he reside on the
land as he is a businessman and storekeeper by occupation and resides at
Lower Sta. Maria, Davao del Sur while the land in litigation is at Colongan,
Sta. Maria. Neither did he put any sign or hut to show that he is in actual
possession. He knew defendants' laborers were in the land in suit as early as
August, 1976 and that they have a hut there but he did not do anything to stop
them. Instead plaintiff was happy that there were people and a hut on the land
in suit.
Therein defendant Ignacio Wong went to the land in litigation to find out if
there were other people residing there or claiming it besides the owner and he
found none. So, in July, 1976, defendant Ignacio Wong bought the parcel of
land in litigation from William Giger and his wife Cecilia Valenzuela (Exhibit 5).
After the execution of Exhibit 5, defendant Ignacio Wong asked for the
delivery of the title to him and so he has in his possession TCT No. (T-4244)
T-974 in the name of William Giger. Mr. Wong declared the land in suit for
taxation purposes in his name. He tried to register the pacto de retro sale with
the Register of Deeds by paying the registration fee but due to some
technicalities, the pacto de retro sale could not be registered. The defendant
Wong placed laborers on the land in suit, built a small farm house after
making some clearings and fenced the boundaries. He also placed
signboards. On September 27, 1976, plaintiff Manuel Mercado again went to
the land in suit to make copras. That was the time the matter was brought to
the attention of the police of Sta. Maria, Davao del Sur and the incident
entered in the police blotter. Wong ordered the hooking of the coconuts from
the land in litigation and nobody disturbed him. But on November 29, 1976,
defendant (Wong) received a copy of plaintiff's complaint for forcible entry
with summons to answer which is the case now before the Court.

66 4S 2014-2015

San Beda College of Law CIVIL LAW REVIEW



The CFI finds the plaintiff (Mercado) to have taken possession of the property
earlier in point of time and defendant (Wong) is an intruder and must, as he is
hereby ordered to return, the possession of the land in question for the
plaintiff, paying a monthly rental of P400.00 from August, 1976, till the
property is returned with costs against the defendant. Judgment is reversed.
Herein petitioner contends that respondent has not established prior
possession and that private respondent's periodic visit to the lot to gather
coconuts may have been consented to and allowed or tolerated by the owner
thereof for the purposes of paying an obligation that may be due to the person
gathering said nuts and that a person who enters a property to gather coconut
fruits and convert the same to copras may only be a hired laborer who enters
the premises every harvest season to comply with the contract of labor with
the true owner of the property.
ISSUES:
1. Is the petitioner a possessor in good faith so as to allow him avoid paying
rentals to private respondent?
2. Is the petitioner correct in stating that respondent did not have prior
possession of the property?
RULING:
1. NO. A perusal of the records of the case shows that petitioner received
private respondent's complaint for forcible entry with summons on November
29, 1976. His good faith therefore ceased on November 29,1976. Accordingly,
the computation of the payment of monthly rental should start from December,
1976, instead of August, 1976. It should be noted that possession acquired in
good faith does not lose this character except in the case and from the
moment facts exist which show that the possessor is not unaware that he
possesses the thing improperly or wrongfully. (Art. 528, Civil Code).
Possession in good faith ceases from the moment defects in the title are
made known to the possessors, by extraneous evidence or by suit for
recovery of the property by the true owner. Whatever may be the cause or the
fact from which it can be deduced that the possessor has knowledge of the
defects of his title or mode of acquisition, it must be considered sufficient to
show bad faith.
2. YES. It should be stressed that "possession is acquired by the material
occupation of a thing or the exercise of a right, or by the fact that it is subject
to the action of our will, or by the proper acts and legal formalities for
acquiring such right." (Art. 531, Civil Code; Rizal Cement Co., Inc. v. Villareal,
135 SCRA 15 [1985]); and that the execution of a sale thru a public
instrument shall be equivalent to the delivery of the thing, unless there is
stipulation to the contrary . . . . If, however, notwithstanding the execution of
the instrument, the purchaser cannot have the enjoyment and material
tenancy of the thing and make use of it herself, because such tenancy and
enjoyment are opposed by another, then delivery has not been effected.
(Paras, Civil Code of the Philippines, Vol. II, 1989 Ed., p. 400).
67 4S 2014-2015

San Beda College of Law CIVIL LAW REVIEW



Applying the above pronouncements on the instant case, it is clear that
possession passed from vendor William Giger to private respondent Manuel
Mercado by virtue of the first sale a retro, and accordingly, the later sale a
retro in favor of petitioner failed to pass the possession of the property
because there is an impediment the possession exercised by private
respondent. Possession as a fact cannot be recognized at the same time in
two different personalities except in the cases of co-possession. Should a
question arise regarding the fact of possession, the present possessor shall
be preferred; if there are two possessions, the one longer in possession, if the
dates of possession are the same, the one who presents a title; and if these
conditions are equal, the thing shall be placed in judicial deposit pending
determination of its possession or ownership through proper proceedings (Art.
538, Civil Code).
SOMODIO v. COURT OF APPEALS
G.R. No. 82680, 15 August 1994
[ALON]
FACTS:
Somodio, petitioner herein, owned a lot numbered 6328-X, over which he
planted ipil-ipil trees, coconut trees and other fruit-bearing trees. In 1976, he
started the construction of a building on the property. The construction was,
however, unfinished because he had to leave for Kidapawan for his
employment and visited the property only intermittently. He allowed
respondent Ayco, to transfer his hut to petitioner's lot. About six years later,
petitioner demanded that Ayco vacate the premises but such demand proved
futile. Hence, Samodio filed an action for unlawful detainer with damages
against respondent Ayco. Meanwhile, the other respondent Purisima entered
the land and constructed a house thereon claiming that Lot No. 6328-X was in
payment of the fee for the services of his father, for having father surveyed
the land and that he caused the construction of a perimeter wall in the area.
Somodio, thereafter, filed against respondent Purisima a complaint for forcible
entry before the same court. The MTC rendered a decision finding that
respondent Purisima built his house "almost on the spot where Somodio's
unfinished house" stood "thru stealth and strategy," not knowing that the
house was built on Lot No. 6328-X and not on Lot No. 6328-Y Subsequently,
the court did not believe respondent Ayco's claim that the administratrix of the
estate of respondent Purisima's father authorized him to build a hut on Lot No.
6328-X in 1976. RTC affirmed in toto the decision of the MTC. The CA,
however, held that herein petitioner had not "clearly and conclusively
established physical, prior possession over Lot No. 6328-X."
ISSUE: Did Somodio establish clearly and conclusively his physical and prior
possession over Lot No. 6328-X?
RULING:
YES. Possession in the eyes of the law does not mean that a man has to
have his feet on every square meter of ground before it can be said that he is
68 4S 2014-2015

San Beda College of Law CIVIL LAW REVIEW



in possession. It is sufficient that petitioner was able to subject the property to
the action of his will.
Article 531 of the Civil Code of the Philippines provides:
Possession is acquired by the material occupation of a thing or the exercise of a right, or by
the fact that it is subject to the action of our will, or by the proper acts and legal formalities
established for acquiring such right.

Even if the Court of Appeals is correct in its finding that petitioner started
introducing improvements on the land only in 1981, he still enjoyed priority of
possession because respondent Purisima entered the premises only in 1983.
Neither did he present proof that between 1958, when his father allegedly
took possession of the land, and 1983, when said respondent himself entered
the land, his father ever exercised whatever right of possession he should
have over the property. Under these circumstances, priority in time should be
the pivotal cog in resolving the Issue of possession.
Petitioner's prior possession over the property, however, is not synonymous
with his right of ownership over the same. As earlier stated, resolution of the
issue of possession is far from the resolution of the issue of ownership.
Forcible entry is merely a quieting process and never determines the actual
title to an estate.
GAVINA MAGLUCOT-AW, CATALINA ORCULLO, RICHARD ESTANO,
NIDA MAGLUCOT, MELANIA MAGLUCOT-CATUBIG, EMILIANO
CATUBIG, LADISLAO SALMA v. LEOPOLDO MAGLUCOT, SEVERO
MAGLUCOT, WILFREDA MAGLUCOT-ALEJO AND CONSTANCIO ALEJO
G.R. No. 132518, 28 March 2000
[BELARMINO]
FACTS:
Petitioners filed with the RTC a complaint for recovery of possession and
damages alleging that they are the owners of Lot No. 1639-D. Said lot was
originally part of Lot No. 1639 which was covered by OCT No. 6775 issued in
the names of Hermogenes Olis, Bartolome Maglucot, Pascual Olis, Roberto
Maglucot, Anselmo Lara and Tomas Maglucot. Tomas Maglucot, one of the
registered owners and respondents predecessor-in-interest, filed a petition to
subdivide the said Lot. Consequently, the CFI of Negros Oriental issued an
order directing the parties to subdivide the lot into 6 portions.
Sometime in 1963, Guillermo Maglucot rented a portion of Lot No. 1639-D.
Subsequently, Leopoldo and Severo, both surnamed Maglucot, rented
portions of the same lot in 1964 and 1969, respectively. In December 1992,
however, the said respondents stopped paying rentals claiming ownership
over the subject lot. Petitioners thus filed a complaint for the recovery of
possession of Lot No. 1639-D and damages.
Petitioners assert that respondents are estopped from claiming to be coowners of the subject lot in view of the mutual agreement in 1946, judicial
69 4S 2014-2015

San Beda College of Law CIVIL LAW REVIEW



confirmation in 1952, and respondents acquiescence because they
themselves exclusively exercised ownership over Lot No. 1639-A beginning
1952 up to the present.
For their part, respondents posit three points in support of their position. First,
they emphasize that petitioners failed to show that the interested parties were
apprised or notified of the tentative subdivision contained in the sketch and
that the CFI subsequently confirmed the same. Second, they point to the fact
that petitioners were unable to show any court approval of any partition. Third,
they maintain that Lot No. 1639 remain undivided since to date, OCT No.
6275 is still an existing and perfectly valid title, containing no annotation of
any encumbrance or partition whatsoever.
The lower court rendered judgment in favor of petitioners. The RTC found the
existence of tax declarations in the names of Hermogenes Olis and Pascual
Olis (purported owners of Lot Nos. 1639-A and 1639-B, respectively) as
indubitable proof that there was a subdivision of Lot No. 1639. It likewise
found that Tomas Maglucot, respondents predecessor-in-interest, took active
part in the partition as it was he, in fact, who commenced the action for
partition.The court a quo cited Article 1431 of the Civil Code which states that
"[t]hrough estoppel an admission or representation is rendered conclusive
upon the person making it, and cannot be denied or disproved as against the
person relying thereon."
ISSUE: Whether parties to a partition proceeding, who elected to take under
partition, and who took possession of the portion allotted to them, are
estopped from questioning the title to partition alloted to another party
RULING:
YES. Parties to a partition proceeding, who elected to take under partition,
and who took possession of the portion allotted to them, are estopped to
question title to portion allotted to another party. A person cannot claim both
under and against the same instrument. In other words, they accepted the
lands awarded them by its provisions, and they cannot accept the decree in
part, and repudiate it in part. They must accept all or none. Parties who had
received the property assigned to them are precluded from subsequently
attacking its validity of any part of it. Here, respondents, by themselves and/or
through their predecessors-in-interest, already occupied of the lots in
accordance with the sketch plan. This occupation continued until this action
was filed. They cannot now be heard to question the possession and
ownership of the other co-owners who took exclusive possession of Lot 1639D also in accordance with the sketch plan.
CEQUENA V. BOLANTE
G.R. No. 137944, 6 April 2000
FACTS:
The land subject of the case was formerly declared for taxation purposes in
the name of Sinforoso Mendoza prior to 1954 but is now declared in the name
70 4S 2014-2015

San Beda College of Law CIVIL LAW REVIEW



of Margarito Mendoza. Petitioners are the daughters of Margarito Mendoza
while the respondent is the only daughter of Sinforoso Mendoza. Margarito
Mendoza and Sinforoso Mendoza were brothers, now deceased. During the
cadastral survey of the property on October 15, 1979 there was already a
dispute between Honorata M. Bolante and Miguel Mendoza, brother of
petitioners. Respondent was occupying the property in question.
ISSUE: Who is the lawful owner and possessor of the land subject of the case
RULING:
The respondents. Despite their dispossession in 1985, the petitioners did not
lose legal possession because possession cannot be acquired through force
or violence. To all intents and purposes, a possessor, even if physically
ousted, is still deemed the legal possessor. Indeed, anyone who can prove
prior possession, regardless of its character, may recover such possession.
Respondent's possession was not disturbed until 1953 when the petitioners'
father claimed the land. But by then, her possession, which was in the
concept of owner -- public, peaceful, and uninterrupted-- had already ripened
into ownership. Furthermore she herself, after her father's demise, declared
and paid realty taxes for the disputed land. Tax receipts and declarations of
ownership for taxation, when coupled with proof of actual possession of the
property, can be the basis of a claim for ownership through prescription.
In contrast, the petitioners, despite thirty-two years of farming the subject
land, did not acquire ownership. It is settled that ownership cannot be
acquired by mere occupation.
Unless coupled with the element of hostility toward the true owner, occupation
and use, however long, will not confer title by prescription or adverse
possession. Moreover, the petitioners cannot claim that their possession was
public, peaceful and uninterrupted. Although their father and brother arguably
acquired ownership through extraordinary prescription because of their
adverse possession for thirty-two years (1953-1985), this supposed
ownership cannot extend to the entire disputed lot, but must be limited to the
portion that they actually farmed.

POSSESSION: Loss of Possession


ARAGON v. THE INSULAR GOVERNMENT
G.R. No. L-6019, 25 March 1911
[BONAVENTE]

FACTS:
In 1892 a possessory title to the land in question was duly registered in favor
of Inocencio Aragon, one of the predecessors in interest of the applicants
Juan Aragon, et.al. For a long period of years, the applicants and their
predecessors in interest have been in possession of the parcel of land in
question, under and undisputed claim of ownership. It is located toward the
71 4S 2014-2015

San Beda College of Law CIVIL LAW REVIEW



center of one of the most valuable residential sections of the city of Manila,
and that for many years a house stood upon this land, and was occupied by
some of the predecessors in interest of the applicants in these proceedings.
There are strong reasons to believe that the land in question was originally
well above the ebb and flow of the tide and that only in later years have the
waters risen to such a height along the shores of the Bay of Manila at this
point as to cover the land in question completely at high tide, though it does
not definitely appear whether this is due to changes in the current and flow of
the waters in the bay, or to the gradual sinking of the land along the coast.
The Court of Land Registration adjudicated title over the land in favor of the
applicants and ordering its registry in accordance with the provisions of "The
Land Registration Act." The Government of the Philippine Islands , through its
proper representatives, objected to the application for registry on the ground
that, as it alleges, the land in question is a part of the public domain. As
alleged by the opponent, the land in question, at the time when the trial was
had in the court below, was so located that at high tide it was completely
covered by the waters of the Bay of Manila, though the receding waters left it
completely bare at low tide.
ISSUE: Did the owners lose their possession of the land in question as
provided under Article 555 of the New Civil Code?
RULING:
NO. If the Government is justified in disturbing the possession of the
applicants, it can only be on the ground that they have abandoned their
property, or that it has been totally destroyed and has now become a part of
the public domain by the erosive action of the sea. It is quite clear that
applicants have never abandoned their possession under a claim of
ownership of this land. There has been no such destructive or total loss of the
property as would justify a holding that the owners have lost possession.
Doubtless the property has been injured by the erosive action of the sea.
Doubtless the owners in order to profitably enjoy the possession of this
property will be compelled to make some relatively small expenditures by way
of a "fill" or a retaining wall. If the applicants have not lost their right of
possession, the Government's claim of ownership, on the ground that this is a
part of the playa (shore) of Manila Bay, necessarily falls to the ground. It
affirmatively appears that the owners of the land in question have never in fact
nor in intent abandoned it, and that keeping in mind its location and actual
condition it can not be said to have been totally destroyed for the purposes for
which it was held by them, so as to have become a part of the playa (shore) of
the Bay of Manila.

72 4S 2014-2015

San Beda College of Law CIVIL LAW REVIEW



CATHOLIC VICAR APOSTOLIC OF THE MOUNTAIN PROVINCE v.
COURT OF APPEALS
G.R. NO. 80294-95, 21 September 1988
[CABBUAG]
FACTS:
The whole controversy started when the herein petitioner filed an application
for registration of lands 1, 2, 3 and 4 in La Trinidad, Benguet on September 5,
1962. The heirs of Juan Valdez and the heirs of Egmidio Octaviano filed an
opposition on lots 2 and 3, respectively. On November 17, 1965, the land
registration court confirmed the registrable title of the petitioner. On May 9,
1977, the Court of Appeals reversed the decision and dismissed the Vicars
application. The heirs filed a motion for reconsideration, praying that the lots
be ordered registered under their names. The Court of Appeals denied the
motion for lack of sufficient merit. Both parties then came before the Supreme
Court. The Supreme Court, in a minute resolution, denied both petitions. The
heirs filed the instant cases for the recovery and possession of the lots.
Respondents argue that the petitioner is barred from setting up the defense of
ownership or long and continuous possession by the prior judgment of the
Court of Appeals under the principle of res judicata. Petitioner contends that
the principle is not applicable because the dispositive portion of the judgment
merely dismissed the application for registration.
ISSUE: Did ownership of Lots 2 and 3 transfer to petitioner Vicar through
acquisitive prescription?
RULING:
NO. Private respondents were able to prove that their predecessors' house
was borrowed by petitioner Vicar after the church and the convent were
destroyed. They never asked for the return of the house, but when they
allowed its free use, they became bailors in commodatum and the petitioner
the bailee. The bailees' failure to return the subject matter of commodatum to
the bailor did not mean adverse possession on the part of the borrower. The
bailee held in trust the property subject matter of commodatum. The adverse
claim of petitioner came only in 1951 when it declared the lots for taxation
purposes. The action of petitioner Vicar by such adverse claim could not ripen
into title by way of ordinary acquisitive prescription because of the absence of
just title.

73 4S 2014-2015

San Beda College of Law CIVIL LAW REVIEW


POSSESSION: Theory
Exceptions Thereto

of

Irrevindicability

and


EDCA PUBLISHING & DISTRIBUTING CORP v. SANTOS
G.R. No. 80298, 26 April 1990
[CRON]
FACTS:
Jose Cruz ordered 406 books from EDCA through telephone, payable on
delivery. Cruz issued a personal check covering the purchase price for the
books. Cruz sold 120 books to Santos who, upon verifying the sellers
ownership from the invoice he showed her, paid him the purchase price.
Upon investigation, EDCA confirmed that Cruz was an impostor and had no
intention of paying the books ordered (the check issued bounced). Thus, with
the assistance of police, EDCA forcibly seized the books from Santos and
threatened her with prosecution for buying stolen property.
Santos sued for the recovery of the books after demand for their return was
rejected by EDCA. EDCA argued that pursuant to Article 559 of the Civil
Code, it has the right to recover the books since it was unlawfully deprived
thereof.
ISSUE: Was EDCA unlawfully deprived of the books because the check issued
by the impostor in payment therefor was dishonored?
RULING:
NO. There was no unlawful deprivation of property which would entitle the
petitioner to recover a property from the person possessing it in good faith.
Actual delivery of the books having been made, Cruz acquired ownership over
the books which he could then validly transfer to the private respondents. The
fact that he had not yet paid for them to EDCA was a matter between him and
EDCA and did not impair the title acquired by the private respondents to the
books.
Non-payment only creates a right to demand payment or to rescind the
contract, or to criminal prosecution in the case of bouncing checks. But absent
the stipulation above noted, delivery of the thing sold will effectively transfer
ownership to the buyer who can in turn transfer it to another.
DE GARCIA v. COURT OF APPEALS
G.R. No. L-20264, 30 January 1971
[CAINDAY]
FACTS:
Guevarra was the owner of a ladys diamond ring. It was stolen from her
house. Later on, while she was talking to Garcia, an owner of a restaurant,
74 4S 2014-2015

San Beda College of Law CIVIL LAW REVIEW



she recognized the ring worn by Garcia as her lost ring. Garcia allege to have
bought it from her comadre. After ascertaining that the ring was hers,
Guevarra demanded the same from Garcia but Garcia refused to return the
same.
ISSUE: What can be a valid defense of a possessor of a thing which is alleged
to have been stolen or lost, against the owner who wishes to recover the
same?
RULING:
One who has lost or has been unlawfully deprived of a movable may recover
the same from the person in possession of the same and the only defense the
latter may have is if he has acquired it in good faith at a public sale in which
case the owner cannot obtain its return without reimbursing the price paid
therefore.
DIZON v. SUNTAY
G.R. No. L-30817, 29 September 1972
[DIVINO]
FACTS:
Respondent Lourdes Suntay is the owner of a three-carat diamond ring
valued at P5,500.00. On June 13, 1962, respondent and Clarita R. Sison
entered into a transaction wherein the respondent's ring was delivered to
Sison for sale on commission. Upon receiving the ring, Sison executed and
delivered to the respondent a receipt. After the lapse of a considerable time
without Sison having returned to the respondent the latter's ring, respondent
made demands on Sison for the return of her ring but the latter could not
comply with the demands because, without the knowledge of the respondent,
on June 15, 1962 or three days after the ring above-mentioned was received
by Sison, said ring was pledged by Melia Sison, niece of the husband of
Clarita R. Sison, evidently in connivance with the latter, with petitioner
Dominador Dizon's pawnshop for P2,600.00. The lower court rendered
judgment declaring that respondent Suntay had the right to the possession of
the ring in question. The Court of Appeals affirmed said judgement. Hence,
the petition for review with the Supreme Court.
ISSUE: Can Suntay recover the diamond ring?
RULING:
YES. The controlling provision is Article 559 of the Civil Code. Respondent
Suntay, having been unlawfully deprived of the diamond ring in question, was
entitled to recover it from petitioner who was found in possession of the same.
The only exception the law allows is when there is acquisition in good faith of
the possessor at a public sale, in which case the owner cannot obtain its
return without reimbursing the price. The right of the owner cannot be
defeated even by proof that there was good faith in the acquisition by the
possessor. The common law principle that were one of two innocent persons
must suffer by a fraud perpetrated by another, the law imposes the loss upon
75 4S 2014-2015

San Beda College of Law CIVIL LAW REVIEW



the party who, by his misplaced confidence, has enabled the fraud to be
committed, cannot be applied in a case which is covered by an express
provision of the new Civil Code, specifically Article 559. Between a common
law principle and a statutory provision, the latter must prevail in this
jurisdiction.
LEDESMA v. COURT OF APPEALS
G.R. No. 86051, 1 September 1992
[DOMINGO]
FACTS:
On September 27, 1977, a person representing himself to be Jojo Consunji,
purchased purportedly for his father, a certain Rustico T. Consunji, two brand
new motor vehicles(Isuzu Gemini and Holden Premier Vehicle) from Citiwide
Motors, Inc. Citiwide Motors delivered the two-above described motor vehicles
to the person who represented himself as Jojo Consunji, allegedly the son of
the purported buyer, Rustico T. Consunji, and said person in turn issued a
managers check of the PCIB for the amount of P101,000.00 as full payment
of the value of the two motor vehicles.However, when Citiwide
Motors deposited the said check, it was dishonored by the bank on the
ground that it was tampered with, the correct amount of P101.00 having been
raised to P101,000.00 per the banks notice of dishonor. Citiwide Motors
reported to the Philippine Constabulary the criminal act perpetrated by the
person who misrepresented himself as Jojo Consunji and in the course of the
investigation, Citiwide Motors learned that the real identity of the
wrongdoer/impostor is Armando Suarez who has a long line of criminal cases
against him for estafa using his similar modus operandi. On October 17, 1977,
plaintiff-appellant was able to recover the Holden Premier vehicle which was
found abandoned somewhere in Quezon City. On the other hand, Citiwide
Motors learned that the Isuzu Gemini was transferred by Armando Suarez to
third persons and was in the possession of one Jaime Ledesma at the time
plaintiff-appellant instituted this action for replevin on November 16, 1977.
In his defense, Jaime Ledesma claims that he purchased and paid for the
subject vehicle in good faith from its registered owner, one Pedro Neyra, as
evidenced by the Land Transportation Commission Registration Certificate.
After posting the necessary bond in the amount double the value of the
subject motor vehicle, plaintiff-appellant was able to recover possession of the
1977 Isuzu Gemini as evidenced by the Sheriffs Return.
ISSUE: Is Citiwide Motors unlawfully deprived of the cars when it sold the
same to a person who purported to be authorized by the buyer to get hold of
the same on the faith of the managers check issued?
RULING:
YES. A party who has lost any movable or has been unlawfully deprived
thereof can recover the same from the present possessor even if the latter
acquired it in good faith and has, therefore, title thereto for under the first
76 4S 2014-2015

San Beda College of Law CIVIL LAW REVIEW



sentence of Article 559, such manner of acquisition is equivalent to a title.
There are three requisites to make possession of movable property equivalent
to title, namely: (a) the possession should be in good faith; (b) the owner
voluntarily parted with the possession of the thing; and (c) the possession is in
the concept of owner. Undoubtedly, one who has lost a movable or who has
been unlawfully deprived of it cannot be said to have voluntarily parted with
the possession thereof. This is the justification for the exceptions found under
the second sentence of Article 559 of the Civil Code.
There was a perfected unconditional contract of sale between private
respondent and the original vendee. The former voluntarily caused the
transfer of the certificate of registration of the vehicle in the name of the first
vendeeeven if the said vendee was represented by someone who used a
fictitious nameand likewise voluntarily delivered the cars and the certificate
of registration to the vendees alleged representative. Title thereto was
forthwith transferred to the vendee. The subsequent dishonor of the check
because of the alteration merely amounted to a failure of consideration which
does not render the contract of sale void, but merely allows the prejudiced
party to sue for specific performance or rescission of the contract, and to
prosecute the impostor for estafa under Article 315 of the RPC. Thus, the
Court ruled that considering that Ledesma had successfully proven that he
acquired the property in good faith from the vendor and for a valuable
consideration, he was unlawfully divested of the car in his possession.

POSSESSION: Fruits

LEONARDO AZARCON, MANUEL AZARCON and ESTEBAN ABOBO v.


VICTOR EUSEBIO
G.R. No. L-11977, 29 April 1959
[ESTEBAN]

FACTS:
Victor Eusebio and petitioners herein had a dispute over the possession of a
certain parcel of public land. The conflict was ordered to be investigated on
May 25, 1955 by the Director of Lands and again on August 3, 1955 by the
Secretary of Agriculture and Natural Resources.
Before the dispute could be settled, Victor Eusebio filed a complaint in the
Court of First Instance of Nueva Ecija praying that defendants be ordered to
vacate the six hectares occupied by them and pay damages. Defendant
Azarcon answered the complaint alleging that he is in actual possession of a
portion of 24 hectares since 1941 by virtue of a homestead application and
that the lease application of plaintiff is subsequent to said homestead
application of Leonardo Azarcon. He, therefore, prayed that the action be
dismissed.

77 4S 2014-2015

San Beda College of Law CIVIL LAW REVIEW



The answer was filed on June 2, 1954 and on motion of plaintiff dated March
15, 1955, the defendants were declared in default. A motion to set aside the
default was denied, and a judgment by default was entered by the court on
April 26, 1955. It ordered defendants to restore possession of the land to
plaintiff. Having failed to obtain a reconsideration of the above decision,
defendants appealed to the Court of Appeals.
While the case was pending in the Court of Appeals, a writ for the execution
of the judgment of the lower court was issued. Said writ of execution was
furnished to the defendants on October 7, 1955. The evidence shows that in
spite of the receipt by the defendants of the notice of the writ of execution of
October 3, 1955, which writ of execution commanded defendants "to forthwith
remove from said premises and that plaintiff have restitution of the same,"
defendants-appellants nevertheless entered the land to gather palay which
was then pending harvest.
ISSUE: Whether the defendants act of entering the contested land to gather
palay which was then pending harvest was lawful despite receipt of order of
execution commanding them to vacate said premises.
RULING:
YES. The Court gathered further from the record that the rice found on the
disputed land at the time of the service of the order of execution had been
planted by defendants-appellants, who appear to have been in possession of
the land from 1951. While the court order of October 3, 1955 ordered the
defendant-appellant to move out from the premises, it did not prohibit them
from gathering the crop then existing thereon. Under the law a person who is
in possession and who is being ordered to leave a parcel of land while
products thereon are pending harvest, has the right to a part of the net
harvest, as expressly provided by Article 545 of the Civil Code.
ART. 545. If at the time the good faith ceases, there should be any natural or
industrial fruits, the possessor shall have a right to a part of the expenses of
cultivation, and to a part of the net harvest, both in proportion to the time of
the possession.
As the order of execution did not expressly prohibit the defendants-appellants
from gathering the pending fruits, which fruits were the result of their
possession and cultivation of the land, it cannot be said that the defendantsappellants committed an act which is clear violation of the courts' order.
Besides, the defendants-appellants had presented, after receipt of the order
of execution, a motion to set aside the said order of execution, and this motion
to stay execution was granted. Defendants furthermore presented a bond in
accordance with the order of the court and had it approved by the Court of
First Instance. It was perhaps in expectation of this resolution of the court
setting aside the order of execution that defendants-appellants may have felt
justified in entering the land and harvesting the fruits existing thereon.
Again the order of the court setting aside its order to stay execution was
issued in the belief that the defendants-appellants had not presented before
78 4S 2014-2015

San Beda College of Law CIVIL LAW REVIEW



the Court of First Instance of Nueva Ecija and which said court actually
approved). Under the circumstances above stated, the Court is not ready to
conclude that the defendants-appellants can be held to have committed a
clear defiance of the order of the court. Their act in harvesting the pending
fruits was not only justified by law but was not expressly prohibited by the
court's order, and was even ratified when the court ordered the suspension of
the execution. There was, therefore, no open, clear and contumacious refusal
to obey a definite order of the court such as would constitute contempt.
Furthermore, a person who has been ordered to leave certain premises is
ordinarily not prohibited from taking with him his own effects and possession,
unless there is an express prohibition to this effect. No such, prohibition was
contained in the order for the defendants to leave the land. There may have
been a technical violation of an order not to enter the premises, but not of one
prohibiting them from removing anything therefrom. Such technical violation of
the order cannot be considered as one amounting to a defiance of the court's
authority, punishable as contempt.
FELIPA CORDERO (Deceased) MAURO OCAMPO, CASIMIRO OCAMPO
and ELISEA OCAMPO v. VICTORIA P. CABRAL, ALEJANDRO
BERBOSO, DALMACIO MONTAOS and HONORABLE COURT OF
APPEALS
G.R. No. L-36789, 25 July 1983
[FERNANDEZ]
FACTS:
Mr. Gregorio Z. Ocampo, husband of Felipa Cordero and father of the other
petitiones, died on May 17, 1958, and left several properties. Petitioners took
possession of the properties left by him, among others is a riceland. However,
they found out that a portion of the same was possessed by Victoria P.
Cabral, Alejandro Berboso and Dalmacio Montaos.
Petitioners filed a civil case alleging that Victoria P. Cabral continued claiming
to be the owner of the land while her co-defendants continued recognizing her
as the owner thereof instead of the plaintiffs despite demands to vacate the
property. They also claim that due to respondents' occupancy of the
aforementioned portion of land, petitioners failed to realize a yearly harvest of
at least ten (10) cavanes of palay at the rate of P10.00 per cavan, from the
harvest-time of 1958 up to the present. RTC dismissed the complaint.
On appeal, even though the CA found that the disputed piece of land is
registered in the name of the petitioners but because of the supposed oral
sale of the same to the predecessors of the defendants mentioned by the
petitioners on appeal, it affirmed the judgment of the trial court dismissing the
complaint for the recovery of the land.
ISSUES:
1. Are the heirs of the registered owner entitled to the land?
2. May the respondents be held liable for reimbursement of fruits received?

79 4S 2014-2015

San Beda College of Law CIVIL LAW REVIEW



RULING:
1. YES. The Court of Appeals found as a fact that the disputed portion of the
land is admittedly part of the land originally registered in the name of
petitioners predecessor in interest. There should be no question that that title
had become imprescriptible and the original registrant as well as his
successors had the right to vindicate their ownership against anybody else.
2. YES. The respondents, by their own admission, are in possession of the
disputed land. There is no evidence that they were possessors in bad faith.
However, their good faith ceased when they were served with summons to
answer the complaint. As possessors in bad faith from the service of the
summons they "shall reimburse the fruits received and those which the
legitimate possessor could have received.

POSSESSION: Expenses


MARTIN MENDOZA and NATALIO ENRIQUEZ v. MANUEL DE GUZMAN
G.R. No. L-28721, 5 October 1928
[J. FERNANDO]

FACTS:
In a cadastral proceeding, a piece of land was adjudicated in favor of Martin
Mendoza and Natalio Enriquez in equal parts pro indiviso subject to the right
of retention on the part of Manuel de Guzman until he shall have been
indemnified for the improvements existing on the land.
Being unable to come to an agreement as to the amount which should be
allowed for the improvements made on the land, Martin Mendoza and Natalio
Enriquez began an action requesting the court to fix the value of the
necessary and useful expenses incurred by Manuel de Guzman in introducing
the improvements. The trial court resolved the questions presented by holding
that in accordance with the provisions of articles 435 and 454 in relation with
article 361 of the Civil Code, the value of the "indemnization" to be paid to the
defendant should be fixed according to the necessary and useful expenses
incurred by him in introducing "las plantaciones en cuestion."
ISSUE: Is the term indemnizacion pertains to the amount of expenditures
such as necessary and useful expenses?
RULING:
YES. The amount of the "indemnizacion" is the amount of the expenditures
mentioned in articles 453 and 454 of the Civil Code, which in the present case
is the amount of the necessary and useful expenditures incurred by the
defendant. Necessary expenses have been variously described by the
Spanish commentators as those made for the preservation of the thing; as
those without which the thing would deteriorate or be lost; as those that
augment the income of the things upon which they are expanded. Among the
necessary expenditures are those incurred for cultivation, production, upkeep,
80 4S 2014-2015

San Beda College of Law CIVIL LAW REVIEW



etc.. Here the plaintiffs have chosen to take the improvements introduced on
the land and are disposed to pay the amount of the necessary and useful
expenses incurred by the defendant. Inasmuch as the retentionist, who is not
exactly a posessor in good faith with in the meaning of the law, seeks to be
reimbursed for the necessary and useful expenditures, it is only just that he
should account to the owners of the estate for any rents, fruits, or crops he
has gathered from it.
ROBLES AND MARTIN v. LIZZARAGA HERMANOS, et.al.
G.R. No. L-16736, 22 December 1921
[L. FERNANDO]
FACTS:
Anastasia de la Rama died leaving six children, one of whom was Evarista
Robles, and some properties, among which was house No. 4 on Iznart Street
in Iloilo City. The heirs of Anastasia then entered into partnership with
Lizarraga Hermanos (LH) in liquidation and settlement of their accounts, by
virtue of which the court awarded to said partnership the properties left by the
deceased, including the aforesaid house No.4.
Evarista, since before the death of her mother Anastasia, has been with her
husband occupying house No. 4, at the beginning, by permission of her
mother, later on by the consent of her coheirs, and lastly by agreement with
the partnership, LH, to whom it had been awarded, paying to said partnership
P40 monthly as rent of the upper story.
Sometime, LH notified Evarista that the rent would be raised to P60 a month.
Evarista refused to pay the new rate and to vacate the house. LH brought suit
against her for ejectment while Evarista sued LH to recover the value of the
improvements she introduced in house no.4. Evarista claims that LH agreed
to sell her the said house, the deed of sale to be executed later; that by virtue
of this contract she remained in the occupation of the building and made the
improvements; that, as one of the stipulations in the contract of sale of the
estate, Evarista assumed the liability of several encumbrances on the estate
all of which payments were made through LH.
ISSUE: Is Evarista Robles the owner of the aforesaid improvements and has
the right to demand payment of their value?
RULING:
YES. Under Article 453 (now Article 546) of the Civil Code, a possessor in
good faith who makes useful improvements on the estate possessed is
entitled to demand payment of the value thereof and to retain estate until the
expenditures incurred therein are paid to him. Here, the presumption of good
faith in favor of Evarista Robles' possession at the time she made the
improvements on the property was neither disputed nor discussed, but on the
contrary, there is positive evidence sufficient to support the conclusion that
when she made the improvements on the aforesaid building she was
possessing it in good faith. Moreover, the expenditures incurred in these
81 4S 2014-2015

San Beda College of Law CIVIL LAW REVIEW



improvements consisting of the addition of a dining room, kitchen, closet, and
bathroom in the lower and upper stories of the house, and a stable, suitable
as a coach house and dwelling were not necessary inasmuch as without them
the house would have continued to stand just as before, but were useful,
inasmuch as with them the house better serves the purpose for which it was
intended, being used as a residence.
If the improvements are useful and Evarista Robles possession was in good
faith, the conclusion set out in article 453 of the Civil Code, supra, is
inevitable; Evarista Robles is the owner of such improvements, and entitled to
reimbursement therefor, and to retain the building until the same is made
METROPOLITAN WATERWORKS AND SEWERAGE SYSTEM v. COURT
OF APPEALS
G.R. No. L-54526, 25 August 1986
[JAVIER]
FACTS:
The City of Dagupan filed a complaint against the former NAWASA (now
MWSS), for recovery of the ownership and possession of the Dagupan
Waterworks System.
NAWASA interposed as one of its special defenses R.A. 1383 which vested
upon it the ownership, possession and control of all waterworks systems
throughout the Philippines and as one of its counterclaims the reimbursement
of the expenses it had incurred for necessary and useful improvements
amounting to P255,000.00.
The trial court found that useful expenses were made in utter bad faith for
they were instituted after the complaint was filed and after numerous Supreme
Court decisions were promulgated declaring unconstitutional the taking by
NAWASA of the patrimonial waterworks systems of cities, municipalities and
provinces without just compensation.
In support of its claim for removal of said useful improvements, MWSS argues
that the pertinent laws on the subject, particularly Articles 546, 547 and 549 of
the Civil Code of the Philippines, do not definitely settle the question of
whether a possessor in bad faith has the right to remove useful
improvements.
ISSUE: Does a possessor in bad faith have the right to remove useful
improvements?
RULING:
NO. Article 449 of the Civil Code of the Philippines provides that "he who
builds, plants or sows in bad faith on the land of another, loses what is built,
planted or sown without right to indemnity." As a builder in bad faith,
NAWASA lost whatever useful improvements it had made without right to
indemnity

82 4S 2014-2015

San Beda College of Law CIVIL LAW REVIEW



Moreover, under Article 546 of said code, only a possessor in good faith shall
be refunded for useful expenses with the right of retention until reimbursed;
and under Article 547 thereof, only a possessor in good faith may remove
useful improvements if this can be done without damage to the principal thing
and if the person who recovers the possession does not exercise the option of
reimbursing the useful expenses.
The right given a possessor in bad faith is to remove improvements applies
only to improvements for pure luxury or mere pleasure, provided the thing
suffers no injury thereby and the lawful possessor does not prefer to retain
them by paying the value they have at the time he enters into possession
(Article 549, Id.).

83 4S 2014-2015

You might also like